Está en la página 1de 44

Captulo 6

Ecuaciones de recurrencia
En argumentos l
ogicos o en algoritmos, cuando hay que dilucidar o resolver una sucesi
on
de casos, el matematico busca averiguar la estructura com
un y la conexi
on de cada caso con
el anterior. La tecnica es astuta: si podemos reducir cada caso al anterior (con un argumento
general), al nal s
olo nos quedar
a un primer caso por resolver, del que se deducir
an todos.

Esta es la esencia del metodo de inducci


on, al que aqu nos estamos reriendo en el sentido
inductivo en el que habitualmente se encuentra en la pr
actica, y no en el deductivo con el
que ineludiblemente se presenta su argumentacion (como hicimos en la seccion 1.2).
Supongamos que los casos que nos interesan son calculos de cierto tipo, como podran
ser los de evaluar el n
umero de estructuras (conjuntos, quiz
as listas) de tama
no o longitud n
con ciertas caractersticas. Observese que cada caso, que se corresponde con cada valor de n,
da lugar a un c
alculo distinto. Ahora, el proceso por el que reducimos un caso al anterior

dar
a como fruto una ecuaci
on general que liga los respectivos resultados. Estas
son las ecuaciones de (o en) recurrencias, o simplemente recurrencias, a las que dedicamos este captulo.
No son asunto nuevo en este libro: hemos deducido ya recurrencias para coecientes bin
omicos, n
umeros de Stirling, y tambien aparecieron en la discusion sobre la ecacia de algoritmos
como la transformada r
apida de Fourier o el metodo de Strassen, por citar algunos ejemplos.
Comenzaremos este captulo con una secci
on en la que el lector podr
a encontrar un amplio
muestrario de cuestiones y argumentos que nos conducen a ecuaciones en recurrencias, que
hemos clasicado en funcion de cu
anta informaci
on sobre los casos anteriores hace falta
conocer y de cuanta complejidad tienen los casos en s. Esta primera seccion pretende ilustrar
la tecnica artesana de esta forma de argumentar.
Luego explicaremos como resolver ecuaciones de recurrencia, o en otras palabras, como
obtener una f
ormula general de las sucesiones de n
umeros que las verican, centr
andonos en
el caso de las ecuaciones de recurrencia lineales y de coecientes constantes, que son quizas
las mas habituales, y sin duda las m
as sencillas. Tambien esbozaremos algunos argumentos
que se pueden utilizar en ciertas variaciones de estas.
En la u
ltima seccion nos dejaremos deslumbrar por la raz
on aurea, producto de la ecuaci
on
en recurrencias mas famosa y fascinante, la ecuacion de Fibonacci.
La extraordinariamente fructfera tecnica general de las funciones generatrices, que permite, en principio, tratar todo tipo de ecuaciones de recurrencia, habr
a de esperar al captulo 10.
341

342

Captulo 6. Ecuaciones de recurrencia

6.1.

Algunos ejemplos

Presentamos en esta seccion inicial una serie de ejemplos, de naturaleza muy diversa,
en cuyo an
alisis encontraremos un variado muestrario de ecuaciones de recurrencia. Algunos
de ellas podremos ya resolverlas, con trucos ad hoc; la resoluci
on de las restantes habr
a de
esperar a disponer de tecnicas mas generales, como las que expondremos en la seccion 6.2 y
en el captulo 10 dedicado a las funciones generatrices.
Por resolver una ecuaci
on de recurrencia, algo que ya avisamos al lector es en general
tarea harto complicada, entendemos hallar una formula explcita para la sucesion de n
umeros
que verican la recurrencia.
1. En funci
on del caso anterior
Las ecuaciones de recurrencia mas sencillas son aquellas en las que el valor de un cierto
termino de la sucesion depende u
nicamente del valor del termino anterior.
Ejemplo 6.1.1 Queremos contar el n
umero de listas de longitud n 1 formadas con ceros
y unos. Llamemos an a la respuesta, para cada n.
No parece un ejemplo muy apasionante, dado que la regla del producto nos permite obtener
directamente la respuesta: an = 2n , para cada n 1. Abordemos el problema desde otro
punto de vista, planteando una recurrencia entre los n
umeros an . Para construir una lista de
longitud n con ceros y unos tomamos primero una lista de longitud n1 con las caractersticas
nadimos
pedidas (lo podremos hacer de an1 formas distintas). Luego, a cada una de ellas, le a
un 0 o un 1, para obtener as todas las posibles listas de longitud n. Como este procedimiento
asocia a cada posible lista de longitud n 1 dos listas distintas de longitud n, se tendr
a que
()

para n 2.

an = 2 an1

Necesitamos, ademas, un valor inicial, el n


umero de listas de longitud 1: a1 = 2. El problema
on () nos permiten
queda as resuelto en el siguiente sentido: el conocimiento de a1 y la relaci
calcular a2 ; con a2 , aplicando de nuevo (), evaluamos a3 , etc. Pero en este caso, ademas,
podemos resolver la recurrencia y obtener una f
ormula explcita para los an : basta aplicar
reiteradamente la regla para obtener que, para cada n 1,
an = 2 an1 = 2 (2an2 ) = 22 an2 = 23 an3 = = 2n1 a1 = 2n .

umero de regiones del plano que determinan n rectas.


Ejemplo 6.1.2 Llamemos an al n
Estas rectas deben ser tales que por cualquier punto del plano pasen a lo sumo dos de ellas;
y tales que ninguna de ellas sea paralela a ninguna otra.
Calculemos los primeros casos: para n = 1, tenemos una recta, que divide al plano en dos
regiones. Para n = 2, n = 3 y n = 4 aparecen cuatro, siete y once regiones, respectivamente:
n=3

n=2
5

1
3

4
2

1
6
2

7
4

n=4

8
9 5
10

1
6
2

11

(versi
on preliminar 23 de octubre de 2008)

7
4

6.1. Algunos ejemplos

343

Ya tenemos los primeros cuatro valores a1 = 2, a2 = 4,


alisis pictorico no nos va
a3 = 7 y a4 = 11. Pero este an
a llevar mucho m
as alla, pues el dibujo se complica enormemente y corremos el riesgo de olvidarnos alguna re1
gi
on. Toca pensar: jemonos en como pasamos de n = 2 a
2
n = 3. Al a
nadir una recta, y como las dos rectas viejas
3
deben cortar a la nueva, lo que hacemos es a
nadir tres
4
regiones del plano. El paso del caso n = 3 al caso n = 4
nadimos una reces analogo (vease el dibujo): de nuevo a
ta, que corta a las tres existentes y crea cuatro regiones nuevas. La generalizacion de este
argumento nos permite deducir que la regla de recurrencia es
2

an = an1 + n

para cada n 2.

Como conocemos el valor para el primer caa a


=
n
so, a1 = 2, podremos calcularlos todos. El lector a n an1 =
n1
n1
n2
podr
a resolver la recurrencia sin m
as que aplicar
..
..
..
.
.
.
reiteradamente la regla de recursion. Alternativa
a
=
2
a
2
1
mente, como la regla de recurrencia nos dice que
an a1
= 2 + 3 + + (n 1) + n
la diferencia entre dos terminos consecutivos es
justamente el valor del ndice (an an1 = n), podemos escribir todas estas diferencias y
umeros
sumarlas, como hacemos a la derecha. Como a1 = 2 y sabemos sumar los n primeros n
naturales, llegamos a la solucion nal:
an = 2 +

n

j=2

j =1+

n

j=1

j =1+

n(n + 1)
2

para cada n 1.

Ejemplo 6.1.3 Las torres de Hanoi.


Cuenta la leyenda1 que los monjes de un monasterio de
1
2
3
Hanoi miden el tiempo que falta para la llegada del n
del mundo con el siguiente procedimiento: disponen de
tres agujas de diamante, en una de las cuales se apilan
64 discos de oro distintos, ordenados por tama
nos. Cada
segundo mueven un disco de una aguja a otra, y su tarea nalizar
a (y con ella el mundo)
cuando logren transportarlos todos a otra aguja. Pero, atenci
on!, a lo largo del proceso nunca
se puede colocar un disco sobre otro de diametro mas peque
no.
Como prepararnos para el n del mundo supondr
a sin duda un notable ajetreo, vamos
a estimar el tiempo de que dispondremos. Para ello, planteamos el problema ya en general:
umero de movimientos necesario para transportar
tenemos n discos y llamamos an al mnimo n
los n discos desde una aguja a otra. Por ejemplo, a1 = 1, porque nos basta con un movimiento
no argumento: podemos,
para pasar el disco a otra aguja. El c
alculo de a2 requiere ya un peque
1

En realidad, la historia que sigue es el adorno literario que concibi


o el matem
atico frances Lucas en 1883.

(versi
on preliminar 23 de octubre de 2008)

344

Captulo 6. Ecuaciones de recurrencia

por ejemplo, pasar el disco peque


no a otra aguja, luego el grande a la tercera, para nalmente
pasar el peque
no a esta tercera aguja, como en la gura:
-

Como en dos movimientos no se puede hacer, concluimos que la descrita es la mejor estrategia
posible, y que, por tanto, a2 = 3.
Si partimos de tres discos, podemos pasar los dos menores a una segunda aguja (con el
procedimiento anterior, de tres movimientos), luego pasar el mayor a la tercera aguja, para
nalmente llevar los dos discos menores sobre ese disco mayor (de nuevo tres movimientos).
En total, 7 movimientos. Aunque ahora no est
a claro si se puede hacer el trasvase con menos.
El procedimiento esbozado en el caso n = 3 se puede generalizar: si tenemos n discos,
pasamos n 1 a una segunda aguja, luego el mayor disco a la aguja nal y, por u
ltimo,
pasamos los n 1 discos a esa tercera aguja. Es un algoritmo recursivo: el procedimiento
para mover n discos se apoya, dos veces, en el (ya conocido) metodo para mover n 1. Se
deduce entonces que el n
umero mnimo de movimientos para transportar n discos cumple que
an 2 an1 + 1

para cada n 2,

on de
porque con 2an1 + 1 movimientos lo sabemos hacer. Observese que no es una ecuaci
recurrencia, sino una desigualdad. Pero lo que nos gustara es comprobar que, en realidad, la
relacion se cumple con un = en lugar de un . Deduciramos as que nuestra estrategia de
movimientos es la mejor posible. Esto requiere un argumento extra.
Veamoslo: si tenemos n discos, en alg
un momento tendremos que mover el disco mayor,
para lo que necesitaremos haber llevado el resto de los discos a otra aguja, pues debe quedar
una aguja libre. Esto requiere, como mnimo, an1 movimientos. Una vez movido el disco
grande a una aguja, tendremos que mover los restantes n 1 discos sobre el, y esto exige, al
menos, otros an1 movimientos (sea cual sea la estrategia que empleemos). As que
an 2 an1 + 1 ,
para n 2. Reuniendo ambas condiciones, ya podemos armar que, para cada n 2,
an = 2 an1 + 1
La condicion inicial ya la hemos visto, es a1 = 1. La resolvemos por simple aplicacion repetida
de la regla de recurrencia:
an = 2 an1 + 1 = 2 (2 an2 + 1) + 1 = 22 an2 + 2 + 1 = 22 (2 an3 + 1) + 2 + 1
= 23 an3 + 22 + 2 + 1 = 23 (2 an4 + 1) 22 + 2 + 1 = 24 an4 + 23 + 22 + 2 + 1
n1

1 2n
= 2n 1 .
2j =
= = 2n1 a1 + 2n2 + 2n3 + + 2 + 1 =
12
j=0

Del caso n = 64 deducimos que el n del mundo llegar


a dentro de a64 = 264 1 segundos,
esto es, mas de medio billon de a
nos! Parece que, despues de todo, la profeca de los monjes
de Hanoi no debera ser una de nuestras mayores preocupaciones.

(versi
on preliminar 23 de octubre de 2008)

6.1. Algunos ejemplos

345

Ejemplo 6.1.4 Calculemos el valor de la siguiente aparentemente complicada integral2 :



tn et dt ,
para cada n 0.
(n + 1) =
0



El caso n = 0 es muy sencillo: (1) = 0 et dt = et 0 = 1 . Vamos a obtener una
ecuacion de recurrencia para los n
umeros (n), pero no a traves de argumentos combinatorios,
como hasta ahora, sino por medio de manipulaciones algebraicas basadas en el metodo de
integraci
on por partes. Sea ahora n 1 y escojamos
u = tn = du = n tn1 dt
dv = et dt = v = et
para la aplicaci
on de la habitual f
ormula de integraci
on por partes:




tn et dt = tn et  +n
tn1 et dt = n (n) .
(n + 1) =
0
0
  


0
=0

(n)

La ecuaci
on de recurrencia resulta ser
(n + 1) = n (n)

para n 1.

Observese que ahora el coeciente que acompa


na al termino anterior no es una constante.
Pero la resoluci
on es, de nuevo, sencilla, por iteracion:
(n + 1) = n (n) = n(n 1) (n 1) = = n(n 1) . . . 3 2 (1) .
Como el valor inicial era (1) = 1, concluimos que, quiz
as sorprendentemente,

tn et dt = n! .
(n + 1) =
0

Antes de seguir adelante, paremonos a reexionar sobre el procedimiento general seguido en los an
alisis que hemos hecho hasta aqu, pues nos servira, con las modicaciones
que exijan los posteriores ejemplos que vayamos viendo, como protocolo general:
1. empezamos analizando el primer caso, digamos el valor de a1 ;
2. luego buscamos argumentos que nos permitan establecer el valor de cada an en terminos
del correspondiente an1 ;
3. con esto ya tenemos resuelto el problema, pues de a1 y la regla de recurrencia obtenemos
el valor de a2 , luego este nos permite calcular a3 , etc.;
4. pero, cuando sea posible, iremos m
as alla en el an
alisis y resolveremos la recurrencia.
Esto es, obtendremos una f
ormula explcita an = f (n) para cada n 1.
2

Que es un caso particular de la funci


on gamma, denida como (z + 1) =
con Re(z) > 1.

(versi
on preliminar 23 de octubre de 2008)

z
t
0

et dt para cada z C

346

Captulo 6. Ecuaciones de recurrencia

2. En funci
on de los dos casos anteriores
En muchas ocasiones, para determinar el valor de un termino de la sucesion, bastar
a disponer del valor de los dos anteriores. Ser
an normalmente ejemplos en los que nos encontraremos
con una disyuntiva: una opci
on nos llevara al caso anterior, mientras que la otra se escribir
a en
terminos del que esta dos posiciones mas all
a.
umero de listas de longitud n, formadas con ceros y unos,
Ejemplo 6.1.5 Llamemos an al n
que no tienen unos consecutivos.
Nos jamos, por ejemplo, en la u
ltima posici
on. Caben dos posibilidades, que la lista acabe
en 0 o en 1. Las que acaban en 0 se construyen dando una de longitud n 1 que cumpla las
condiciones (ceros y unos, sin unos consecutivos) y a
nadiendole un 0 al nal (compruebese que
es una biyeccion). Las acabadas en 1 llevan, necesariamente, un 0 en la pen
ultima posici
on;
as que habr
a tantas como listas de longitud n 2 que cumplan las condiciones:

0
an1





una de longitud n 1
an

0 1
an2





una de longitud n 2

obligado!

Por tanto, la regla de recurrencia ser


a, para cada n 3,
an = an1 + an2
Una ecuaci
on muy especial, conocida como la ecuaci
on de Fibonacci, que volver
a a aparecer en los siguientes ejemplos. Observemos que los valores iniciales, que permiten arrancar

a la sucesion, son aqu a1 = 2 y a2 = 3.


Ejemplo 6.1.6 Tenemos una escalera con n pelda
nos y en cada paso se permite subir 1
umero de formas distintas de subir la escalera con ese tipo
2 pelda
o
nos. Llamamos an al n
de pasos (dos formas de subir ser
an distintas si la secuencia de pasos seguida es distinta).
Para contar cu
antas de estas peculiares ascensiones de una escalera con n pelda
nos hay, las
clasicaremos atendiendo a como se completa la escalada. Hay dos posibilidades:
llegar al pelda
no n 1 y luego subir el u
ltimo pelda
no en un paso;
o llegar al n 2 y luego un paso de dos pelda
nos.
As que la relaci
on de recurrencia es, de nuevo, an = an1 + an2 para cada n 3. Aunque
ahora los valores iniciales no son los del ejemplo anterior, sino que a1 = 1 y a2 = 2. Como
las condiciones iniciales son distintas, las sucesiones de n
umeros no coinciden, aunque en
realidad una esta desplazada una posici
on de la otra:
a1 a2 a3 a4 a5 a6
# listas del ejemplo 6.1.5 2 3 5 8 13 21
# caminos en la escalera

13

Como veremos en el siguiente ejemplo, el comportamiento asintotico de la sucesion no depende


de los valores iniciales, sino que viene dictado por la ecuaci
on.

(versi
on preliminar 23 de octubre de 2008)

6.1. Algunos ejemplos

347

Ejemplo 6.1.7 Un modelo de cunicultura.


Fibonacci3 introdujo la sucesi
on que lleva su nombre4 como modelo
para la reproducci
on de conejos. Parta Fibonacci de ciertas hip
otesis, a
saber: (a) los conejos viven eternamente; (b) cada mes, un par de adultos
de distinto sexo da lugar a un nuevo par de conejos de distinto sexo; y
(c) cada conejo se hace adulto a los dos meses de vida, momento en el que
umero de pares
comienza a tener descendencia. Designemos por Fn el n
de conejos al nal del mes n. Partimos de un par de conejos que nacen en
el primer mes; esto es, F1 = 1. Al cabo de un mes seguiremos teniendo
una pareja de conejos, todava no adultos: F2 = 1. En el tercer mes
ya tenemos una pareja de adultos, que da lugar a una pareja de recien
nacidos: F3 = 2. En el cuarto mes seguiremos teniendo una pareja de
adultos, que tendr
a descendencia. Y la pareja nacida en el mes anterior
Figura 6.1: Fibonacci
tendr
a ahora un mes. En total, habr
a tres parejas de conejos: F4 = 3.
Y as, sucesivamente.
La tabla de la derecha recoge las
Mes
1 2 3 4 5 6 7
distintas poblaciones al comienzo de
Parejas de adultos
0 0 1 1 2 3 5
umero de parejas en
cada mes. El n
Parejas con un mes de edad 0 1 0 1 1 2 3
el mes n es la suma del n
umero de
Parejas de recien nacidos
1 0 1 1 2 3 5
parejas en el mes n 1 mas las paN
umero total de parejas, Fn 1 1 2 3 5 8 13
rejas que nacen en el propio mes n.
De estas hay tantas como parejas adultas hubiera en el mes n. Y a su vez, tantas como parejas
en el mes n 2 (pues se tardan dos meses en ser adulto). En total, para cada n 3,
Fn = Fn1 + Fn2 ;

de nuevo, la ecuaci
on de Fibonacci.

on de recurrencia es valida para cada n 2. En lo sucesivo


Si denimos F0 = 0, la ecuaci
umeros siguientes:
reservaremos el nombre de Fn para los n
Definici
on 6.1 La sucesi
on de n
umeros de Fibonacci (Fn ) cumple, para cada n 2, que
(la ecuaci
on de Fibonacci)

Fn = Fn1 + Fn2

on son
junto con las condiciones iniciales F0 = 0, F1 = 1. Los primeros terminos de esta sucesi
F0

F1

F2

F3

F4

F5

F6

F7

F8

F9

F10

F11

F12

13

21

34

55

89

144

La ilustraci
on que aqu reproducimos es la cabeza de la estatua que erigieron cerca del ro Arno los
ciudadanos de Pisa en honor de Leonardo de Pisa (1170-1250), tambien llamado Fibonacci (hijo de Bonacci).
Fibonacci viaj
o por todo el Mediterr
aneo y en el curso de sus viajes tom
o contacto con las Matem
aticas que los
arabes haban recopilado. Su Liber Abaci de 1202 tuvo una enorme inuencia en su tiempo. En el se recogan

numerosos resultados sobre Geometra y Teora de N


umeros, adem
as de cuestiones pr
acticas sobre problemas
mercantiles, metodos de multiplicaci
on, etc. El texto estableci
o denitivamente en Occidente los numerales
ar
abigos y el sistema hind
u de numeraci
on.
4
Fue el matem
atico frances Edouard Lucas (1842-1891) el que as la bautiz
o. En el ejemplo 6.3.1 veremos
una sucesi
on de n
umeros que lleva su nombre y que est
a muy relacionada con los de Fibonacci. Lucas trabaj
o fundamentalmente en Teora de N
umeros, con estudios sobre la sucesi
on de Fibonacci o sobre criterios de
primalidad (criterio de Lucas-Lehmer). Tambien gustaba de proponer juegos y acertijos matem
aticos, como el
de las torres de Hanoi que veamos en el ejemplo 6.1.3, que recogi
o en sus Recreations mathematiques de 1882.

(versi
on preliminar 23 de octubre de 2008)

348

Captulo 6. Ecuaciones de recurrencia

Aunque tendremos que esperar a la seccion 6.2 para obtener una f


ormula para estos
n
umeros Fn , podemos ya analizar su comportamiento asintotico; esto es, el orden de magnitud
de los n
umeros Fn cuando n . Observemos, para empezar, los valores numericos de
sucesivas razones de n
umeros de Fibonacci consecutivos:
1/1

2/1

3/2

5/3

8/5

13/8

21/13

34/21

55/34

89/55

144/89

1,5000

1,6666

1,6000

1,6250

1,6153

1,6190

1,6176

1,6181

1,6179

Que parecen indicar que los n


umeros Fn crecen, mas o menos, con una tasa ja, pues el
cociente de dos n
umeros de Fibonacci consecutivos Fn /Fn1 es, aproximadamente, como una
constante x > 1 (en torno a 1,61 y pico) cuando n es grande. En otras palabras, parece que
Fn
= x.
lm
n Fn1
Si fuera cierta esta intuici
on heurstica, que condicion habra de cumplir ese hipotetico n
umero x? Reescribamos la ecuacion de Fibonacci para exhibir cocientes de terminos consecutivos:
Fn
1
=1+
.
Fn = Fn1 + Fn2 =
Fn1
Fn1
Fn2
Pasando al lmite n , el n
umero x debera cumplir que
1
x = 1 + ; o en otras palabras, que x2 x 1 = 0 .
x

Las soluciones de esta ecuacion de segundo grado son los n


umeros 1+2 5 y 12 5 . El segundo
de ellos, un n
umero negativo, no puede ser esa tasa ja de crecimiento, pues los Fn son
positivos. El otro, la raz positiva de la ecuaci
on, es un n
umero sobre el que ya hablamos
1+ 5
en el captulo 1: la raz
on a
urea = 2 , a cuyas asombrosas peculiaridades dedicaremos
toda la seccion 6.3.
Pero pong
amonos serios (y formales): es cierto o no que Fn /Fn1 tiende a cuando
n ? Restamos primero a ambos lados de la ecuacion de Fibonacci, utilizamos que
= 1 + 1/ y reordenamos los terminos astutamente, para obtener que:


1 Fn2 Fn1
Fn2
Fn2
1
Fn
=1+
=
=
.
Fn1
Fn1
Fn1
Fn1 Fn2
Ahora, tomamos valores absolutos:


 1 Fn2
 Fn


 Fn1  = Fn1





 Fn1


 Fn1
1




 Fn2  <  Fn2  ,

donde hemos utilizado que los n


umeros de Fibonacci forman una sucesi
on creciente y que, por
tanto, Fn2 /Fn1 < 1. Observese la simetra de la expresion: a la derecha y a la izquierda
tenemos la diferencia entre un cociente de n
umeros de Fibonacci consecutivos y la raz
on
aurea; pero a la derecha, de ndices mas peque
nos! Iterando esta desigualdad, llegamos a que








 1  Fn1




 Fn2
 F2
 Fn
1
1
1 









 Fn1  <  Fn2  < 2  Fn3  < < n2  F1  = n2 1 .
Como > 1, deducimos que |Fn /Fn1 | 0 cuando n y que, por tanto, Fn /Fn1
cuando n .

(versi
on preliminar 23 de octubre de 2008)

6.1. Algunos ejemplos

349

Ejemplo 6.1.8 Desbarajustes.


En la subsecci
on 3.2.3 nos ocupamos del n
umero Dn de desbarajustes de {1, . . . , n}, las
permutaciones de {1, . . . , n} en las que ning
un smbolo ocupaba su posici
on natural. Obtuvimos entonces una formula (un tanto complicada) para Dn , utilizando el principio de
inclusi
on/exclusi
on. Buscamos ahora una relaci
on de recurrencia para estos n
umeros.
Los primeros valores son f
aciles de calcular, por simple enumeraci
on de casos: D1 = 0,
D2 = 1 y D3 = 2. Analicemos el caso n = 4, que ya contiene los ingredientes del caso general.
Tomemos un desbarajuste de los que cuenta D3 ; por ejemplo,


1 2 3
.
3 1 2
Queremos construir un desbarajuste de cuatro posiciones a partir de el. El elemento 4 no
puede ir en la cuarta posici
on (no sera desbarajuste), as que necesariamente ha de ir en una
de las tres anteriores. Si va, por ejemplo, en la primera, colocaremos en la cuarta posici
on el
elemento que antes iba en la primera (para que siga siendo un desbarajuste); y lo mismo si
va en la segunda o en la tercera:



 
 

1 2 3 4
1 2 3 4
1 2 3 4
1 2 3 4

,
,
3 1 2
4 1 2 3
3 4 2 1
3 1 4 2
As que, para cada desbarajuste de tres posiciones podemos formar 3 desbarajustes de cuatro posiciones (uno por cada posible posible posicion en la que colocar el 4). Sin embargo,
hay unos desbarajustes de cuatro posiciones que no podemos construir de esta manera: por
ejemplo, el siguiente:


1 2 3 4
,
4 3 2 1
pues construir este desbarajuste con el procedimiento anterior requerira haber partido de
una permutaci
on que no es un desbarajuste, la lista (1, 3, 2). Aunque convendr
a el lector que
este desbarajuste de longitud 4 es muy especial: en el, el 4 y el 1 forman un ciclo.
Pero para construir un desbarajuste (de longitud 4) en el que el 4 y otro elemento formen
una transposici
on, basta construir un desbarajuste con el resto de las posiciones (en este caso,
dos). Tenemos, ademas, libertad para elegir el elemento con el que el 4 forma ciclo: puede
ser el 1, el 2 o el 3. De manera que de estos desbarajustes de longitud 4 hay 3D2 . De donde
deducimos que
D4 = 3 D3 + 3 D2 .
Hagamos el argumento en el caso general de un Dn
n
1 2 3 4
cualquiera. Tenemos, por un lado, los desbarajustes
en los que n forma una transposici
on con alg
un otro
n
1



elemento. En el esquema de la derecha, el 1. De estos
tendremos (n 1)Dn2 , porque hay n 1 candidatos
Desbarajuste de n 2 posiciones
a formar la transposici
on con n.
Por otro lado, tendremos los desbarajustes en los que la situaci
on anterior no ocurre, es
decir, en los que el elemento n no forma ciclo de orden 2 con otro elemento. Pero estos son
(versi
on preliminar 23 de octubre de 2008)

350

Captulo 6. Ecuaciones de recurrencia

justamente los desbarajustes que se pueden formar a partir de los de n 1 posiciones con
el siguiente proceso: por cada desbarajuste de n 1 posiciones, elegimos una posici
on i para
colocar el elemento n; en la posici
on n colocamos lo que hubiera en la posici
on i.
1

n1 n

n1 n

De estas habr
a (n 1)Dn1 . En total,
Dn = (n 1)Dn1 + (n 1)Dn2

para n 3.

Note el lector que esta relacion requiere, para obtener el valor de un cierto termino, el
conocimiento de los dos anteriores. Pero, a diferencia de lo que ocurra con los ejemplos
anteriores, los coeficientes no son constantes, sino que dependen del paso en que estemos.
Afortunadamente, podemos encontrar un truco que nos permite reducir esta ecuaci
on a
una que s
olo involucra el termino anterior, que resolveremos luego por simple iteraci
on. La
clave es observar que podemos reescribir la recurrencia como
Dn nDn1 = Dn1 + (n 1)Dn2 = [Dn1 (n 1)Dn2 ] .
En estos nuevos terminos, solo queda iterar la relaci
on:
Dn nDn1 = [Dn1 (n 1)Dn2 ] = (1)2 [Dn2 (n 2)Dn3 ] =
= = (1)n2 [D2 2D1 ] = (1)n2 = (1)n ,
donde hemos utilizado los valores iniciales D2 = 1 y D1 = 0. As que, para cada n 2,
Dn nDn1 = (1)n ,
que, observese, involucra solo el termino anterior. Si dividimos toda la expresion por n!,
Dn1
(1)n
Dn

=
,
n!
(n 1)!
n!
obtenemos una ecuacion que se resuelve con el procedimiento (ya visto en el ejemplo 6.1.2)
de sumar todas estas expresiones desde n hasta 1. As llegamos a que
Dn D1  (1)j

=
.
n!
1!
j!
n

j=2

En otras palabras, como D1 = 0,


Dn = n!

n

(1)j
j=2

j!

= n!

n

(1)j
j=0

j!

porque los terminos j = 0 y j = 1 se cancelan entre s. Es, claro, el mismo resultado que el
que obtenamos con el principio de inclusi
on/exclusi
on.

(versi
on preliminar 23 de octubre de 2008)

6.1. Algunos ejemplos

351

En ocasiones, las condiciones iniciales que determinan, junto a la ecuaci


on de recurrencia, los valores de la sucesion, no son como las que hemos visto hasta aqu.
Ejemplo 6.1.9 La ruina del jugador5 .
Para analizar este ejemplo, al que volveremos en diversas ocasiones a lo largo de este captulo, apelaremos a que el lector est
a familiarizado con ciertos conceptos probabilsticos, en
particular, con la noci
on de probabilidad condicionada (en caso contrario, puede consultarse
primero el captulo 7).
Queremos apostar nuestro dinero en un cierto juego. En cada partida apostamos 1 euro. Si
ganamos, nos devuelven el euro apostado junto con otro m
as, y en caso contrario perdemos el
euro apostado. La probabilidad de ganar cada partida es p, un n
umero entre 0 y 1 (la de perder
sera q = 1 p). Fijamos a priori una cota de N euros, y solo abandonaremos el juego cuando
nuestra fortuna acumulada tras sucesivas partidas la haya alcanzado o cuando nos hayamos
arruinado6 . Supongamos que al empezar a jugar disponemos de n euros (0 n N ). La
pregunta es: cu
al es la probabilidad, que llamaremos a(n), de que nos arruinemos?
Observe el lector que a(0) = 1, porque si la fortuna inicial es 0, no podremos ni siquiera
empezar a jugar. Por otro lado, a(N ) = 0, porque si al principio tenemos N euros, tal y
como hemos convenido, nos retiramos. Como decamos antes, no son estas unas condiciones
iniciales al uso, pero seran sucientes7 .
Supongamos que 0 < n < N . Para obtener la regla de recurrencia que cumplen los a(n),
vamos a condicionar sobre el resultado de la primera partida. Llamemos G al suceso ganar
la primera (cuya probabilidad es p) y P al suceso perder la primera (de probabilidad
q = 1 p). Entonces,
P(ruina) = P(ruina|G) P(G) + P(ruina|P ) P(P ) .
Pero si hemos ganado la primera partida, tendremos n + 1 euros; y la probabilidad de arruinarnos es la misma que si hubieramos empezado a jugar con esos n + 1 euros. El mismo
argumento, hecho para el caso de la perdida de la primera partida, nos lleva a que
a(n) = p a(n + 1) + q a(n 1)

para cada 1 n N 1.

Relacion que conviene escribir como


a(n + 2) =

q
1
a(n + 1) a(n)
p
p

para cada 0 n N 2

Resolveremos este problema (la regla de recurrencia junto con las condiciones a(0) = 1 y
a(N ) = 0) en el ejemplo 6.2.2, donde comprobaremos la extraordinaria importancia que
tiene el que estemos ante un juego equilibrado (p = q = 0,5) o no (p = q).

O es que quiz
as conoce el lector alg
un jugador que no acabe en la ruina?
Acaso podramos estar jugando indenidamente? No es el caso: se puede demostrar, pero esto requiere
el manejo del c
alculo de probabilidades, que el juego acaba, con uno u otro resultado, con probabilidad 1.
7
Quiz
as el lector avezado preferira llamarlas condiciones de contorno o de frontera.
6

(versi
on preliminar 23 de octubre de 2008)

352

Captulo 6. Ecuaciones de recurrencia

3. En funci
on de todos los casos anteriores
Ejemplo 6.1.10 Contemos el n
umero de a
rboles con raz casi binarios que podemos formar
con n vertices y en los que distinguiremos entre derecha e izquierda.
Aunque m
as adelante (en la seccion 8.2.3) los estudiaremos con mas detalle, no es difcil entender lo que
=
es un arbol binario con raz: la raz es el vertice que
situamos (curiosamente) mas arriba, y el que sea (casi)binario quiere decir que cada vertice puede tener hasta dos descendientes hacia abajo.
En cuanto a la diferencia entre izquierda y derecha, el dibujo de la derecha muestra dos
umero de arboles con estas
arboles que nosotros consideraremos como distintos. Sea Cn al n
caractersticas. Los primeros valores son C1 = 1 y C2 = 2. En el siguiente dibujo exhibimos
los cinco arboles distintos que se pueden formar con tres vertices:

As que C3 = 5. Puede el lector entretenerse dibujando los 14 arboles con


cuatro vertices que existen para comprobar que C4 = 14. Argumentemos
en general, para Cn : cualquier arbol de este tipo tendra k vertices a la
k
nk1
izquierda de la raz y n 1 k a la derecha, donde 0 k n 1, como
v
ertices
v
ertices
en el dibujo. Las elecciones del arbol que va a la izquierda de la raz y la
del que va a la derecha son independientes, y para contar el n
umero de posibles elecciones de
ambos tendremos en cuenta la diferencia izquierda-derecha. Aplicando las reglas de la suma
y del producto y deniendo, como resulta conveniente, C0 = 1, obtenemos que
Cn = C0 Cn1 + C1 Cn2 + + Cn2 C1 + Cn1 C0 =

n1


Ck Cn1k

k=0

Ahora, para calcular un cierto termino Cn , necesitamos conocer todos los anteriores. Esta
recurrencia es la que cumplen los n
umeros de Catalan, como ya vimos en el ejemplo 2.3.3.
As que hemos encontrado otro problema combinatorio cuya respuesta est
a en estos n
umeros,
de los que ya obtuvimos una f
ormula explcita (vease el ejemplo 3.1.3). Resolveremos esta
recurrencia utilizando las tecnicas de las funciones generatrices (vease el ejemplo 10.4.5).
Ejemplo 6.1.11 El problema de los montones de barriles.
Miramos, desde un lateral, la carga de un cami
on lleno de barriles de cerveza8 . Nos interesan
las disposiciones de barriles en las que, en cada la, los barriles forman una tira continua; y
cada barril, por una simple cuesti
on de equilibrio, ha de apoyarse en dos de la la inferior:
S es un mont
on de 6 barriles

No lo es

Tampoco

Interesa conocer Mn , el n
umero de montones distintos que tienen n barriles en la la base.
8

Dejamos al lector que proponga la marca, en funci


on de su ideologa cervecera.

(versi
on preliminar 23 de octubre de 2008)

6.1. Algunos ejemplos

353

Por ejemplo, M1 = 1 y M2 = 2. Si n = 3, tenemos los


cinco posibles montones del dibujo. Compruebe el lector
que hay 13 posibles montones con 4 barriles en la base.
Cualquier mont
on con n barriles en la base tiene encima un mont
on con una base de j
barriles, donde 0 j n 1. Una vez elegido j y decidido que tipo de mont
on es (de los Mj
posibles), solo queda colocarlo: si j = 0, no tenemos eleccion (y queda u
nicamente la la
original); pero si j = 0, el lector puede comprobar que hay n j posibles posiciones. As que
Mn = 1 +

n1


(n j)Mj

para cada n 2.

j=1

M
as adelante (vease el ejemplo 10.4.4), y ya con funciones generatrices, deduciremos que Mn
es, simplemente, el n
umero de Fibonacci F2n1 , para cada n 1.

Ejemplo 6.1.12 Los n


umeros de Bell.
El n
umero de Bell B(n) cuenta, como ya vimos en la subseccion 3.3.1, el n
umero total de
particiones de {1, . . . , n} en bloques no vacos: B(1) = 1, B(2) = 2, B(3) = 5, etc. Queremos escribir una recurrencia para B(n + 1) (algo que pedamos hacer en el ejercicio 3.3.3).
Fijemonos en un bloque especial, el que contiene al elemento n + 1. Este bloque estar
a formado, adem
as de por el n + 1, por un cierto subconjunto de {1, . . . , n}. Llamemos n k al
tama
no de ese subconjunto, donde 0 k n (observese que tanto k = 0 como k = n estan
n
maneras distintas.
permitidos). Un subconjunto de ese tama
no puede ser elegido de nk
Una vez decididos que elementos acompa
nan a n + 1 en su bloque, basta partir en bloques
los k elementos restantes. De todo esto se deduce que, para cada n 1,

n 
n  


n
n
B(n + 1) =
B(k) =
B(k) para cada n 0.
B(n + 1) = 1 +
nk
k
k=1

k=0

A la derecha hemos decidido, por convenio (y comodidad), que B(0) = 1.

4. Mirando lejos
En los ejemplos vistos hasta aqu, el calculo de un termino de la sucesion requiere el
conocimiento del anterior, de los dos anteriores. . . e incluso de todos los anteriores. Pero a
veces la recurrencia es mas caprichosa, y exige buscar el termino que este un cierto n
umero
de posiciones antes, o todas las de un cierto bloque anterior de la sucesi
on.
Ejemplo 6.1.13 Un algoritmo para calcular el m
aximo y el mnimo de n n
umeros.
La estrategia que seguiremos se inscribe dentro de los procedimientos de divide y vencer
as,
de los que ya hemos visto alg
un ejemplo anteriormente: las multiplicaciones de n
umeros o
matrices de las secciones 4.5 y 4.4, o la Transformada rapida de Fourier de la subsecci
on 4.6.5.
Partimos de una lista de n n
umeros (a1 , . . . , an ) y queremos determinar el mayor y el menor de
ellos utilizando comparaciones entre pares de n
umeros. Para cada n 2, vamos a llamar Cn
al n
umero de comparaciones necesarias para determinar el maximo y el mnimo de los n
n
umeros siguiendo el procedimiento que pasamos a describir.
(versi
on preliminar 23 de octubre de 2008)

354

Captulo 6. Ecuaciones de recurrencia

Vamos a analizar primero el caso en el que n es un n


umero par. El primer caso, algo
on para determinar cu
al de los dos
especial, es C2 = 1, pues basta con hacer una comparaci
n
umeros es mayor y cual menor. Si n es un n
umero par 4, partimos la lista exactamente
por la mitad. Para obtener el m
aximo y el mnimo de los n n
umeros, determinamos primero
el maximo y el mnimo de cada uno de los bloques, y luego comparamos el m
aximo de los
dos bloques (una comparaci
on) y el mnimo de los dos (otra comparaci
on). As que
para cada n par 4.

Cn = C n2 + C n2 + 2

Para el caso de longitud n impar, de nuevo n = 3 es especial, y el resultado es C3 = 3. El


argumento general, para n = 2k + 1 5 es como el de antes, con la salvedad de que ahora
debemos cortar en un bloque de k n
umeros y otro de k + 1. Si el lector completa el argumento
para este caso y re
une los dos, llegara a la f
ormula general siguiente:
para cada n 4,

Cn = C n + C n  + 2
2
2

que junto con los valores C2 = 1 y C3 = 3 permite obtener la sucesion (Cn ) completa.

Ejemplo 6.1.14 Los n


umeros de Leibniz.
Escribimos los n
umeros naturales en base 2:
0

10

11

12

13

14

15

10

11

100

101

110

111

1000

1001

1010

1011

1100

1101

1110

1111

y sumamos los dgitos de esos desarrollos binarios. Llamemos L(n) al resultado de esas sumas:
n

10

11

12

13

14

15

L(n)

Se cuenta que fue Leibniz el que primero se j


o en la sucesion (L(n))
mientras esperaba a ser recibido en audiencia por el Papa, a quien iba
a plantear una propuesta para unicar las distintas iglesias cristianas9 .
Observese que si, para un cierto k, n < 2k , entonces los desarrollos
binarios de n y n + 2k coinciden excepto en la primera posicion: n + 2k
llevara un 1 y a n le podemos poner un 0. De manera que
L(n + 2k ) = L(n) + 1

Figura 6.2: Leibniz

si 0 n < 2k . As que para calcular el valor de L(n + 2k ) necesitamos mirar al n


umero L(n),
que est
a mucho antes en la sucesion.
9

Los matem
aticos de anta
no tenan intereses muy diversos. En todo caso, curiosa manera de preparar
una entrevista para un asunto de tan aparente trascendencia. Gottfried Leibniz (1646-1716), el matem
atico y
l
osofo alem
an protagonista de este chascarrillo, es considerado, junto con Newton, el inventor del C
alculo.
Leibniz invent
o notaciones que han llegado a nuestros das, como dx o el smbolo , y reglas (que hoy llevan
su nombre) como la de diferenciaci
on de productos. Para 1676 haba completado su formulaci
on del C
alculo,
incluyendo el Teorema Fundamental. Newton haba desarrollado su metodo de uxiones en 1671, aunque su
trabajo no fue publicado hasta 1736, y acus
o a Leibniz de plagio. Una polemica hist
orica: quien fue antes?
Fuera quien fuera, parece claro que ambos llegaron a sus conclusiones de manera independiente y que, por
supuesto, ambos merecen una posici
on destacada en el Olimpo matem
atico.

(versi
on preliminar 23 de octubre de 2008)

6.1. Algunos ejemplos

355

Veamos como se generan recursivamente los valores de la sucesion: los vamos a agrupar
dependiendo del bloque di
adico [2k , 2k+1 ) al que pertenezcan. Empezamos con
(L(0), L(1)) = (0, 1)
y entonces

 
 
      
L(2), L(3) = L(0 + 2), L(1 + 2) = L(0) + 1, L(1) + 1 = 0, 1 + 1, 1 = 1, 2 .
El siguiente bloque se obtendra

 

L(4),L(5), L(6), L(7) = L(0 + 4), L(1 + 4), L(2 + 4), L(3 + 4)
 


 
 
= L(0) + 1, L(1) + 1, L(2) + 1, L(3) + 1 = 0, 1, 1, 2 + 1, 1, 1, 1 = 1, 2, 2, 3 .
El procedimiento es sencillo: para calcular los de un bloque di
adico, tomamos la lista de todos
los anteriores y les sumamos, a cada uno de ellos, 1. Por ejemplo, el siguiente bloque sera
(L(8), L(9), L(10), L(11), L(12), L(13), L(14), L(15)) =
= (L(0), L(1), L(2), L(3), L(4), L(5), L(6), L(7)) + (1, 1, 1, 1, 1, 1, 1, 1) = (1, 2, 2, 3, 2, 3, 3, 4).
Observese que estos n
umeros cumplen ademas que
L(2n) = L(n) ,
porque multiplicar por 2 signica rodar los coecientes del desarrollo binario de n una unidad,
a
nadir un cero a la derecha: por ejemplo, 7 = (101)2 y 14 = (1010)2 ; 31 = (11111)2 y
on (Ln ).
62 = (111110)2 . La identidad anterior es tambien una recurrencia para la sucesi
5. Ecuaciones con dos par
ametros
En el captulo 3 analizamos diversas familias de n
umeros que dependen de dos par
ametros: coecientes binomicos, n
umeros de Stirling de primera y segunda especie, n
umero de
particiones de un entero en un cierto n
umero de partes, etc. Para cada uno de ellos, encontramos relaciones de recurrencia y valores iniciales que permitan calcularlos. Por recordarlos,
veamos un par de casos.
Ejemplo 6.1.15 Coeficientes bin
omicos y n
umeros de Stirling de segunda especie.
El n
umero de subconjuntos de tama
no k que se pueden extraer del conjunto {1, . . . , n},
C(n, k), satisface la recurrencia doble
C(n, k) = C(n 1, k) + C(n 1, k 1)
para cada n 1 y para cada k 1. Las condiciones iniciales son las de los bordes del triangulo
de Tartaglia: C(n, 0) = 1, para todo n 0 y C(n, n) = 1, para todo n 0. Disponemos,
ademas, de una f
ormula cerrada para estos n
umeros:
 
n
n!
,
C(n, k) =
=
k! (n k)!
k
v
alida para cada n, k 0. Sobre lo ecaz que resulta el c
alculo con la f
ormula o con la
recurrencia ya reexionamos en la subseccion 3.1.1.
(versi
on preliminar 23 de octubre de 2008)

356

Captulo 6. Ecuaciones de recurrencia

Cuando cont
abamos el n
umero de particiones en k bloques no vacos del conjunto {1, . . . , n},
obtenamos los n
umeros de Stirling de segunda especie, S(n, k), que satisfacan la relaci
on
S(n, k) = kS(n 1, k) + S(n 1, k 1)
junto con los valores frontera S(n, n) = 1 y S(n, 1) = 1. Tambien en esta ocasion tenemos
una f
ormula:
 
k1
1 
k
S(n, k) =
(1)j
(k j)n ,
k!
j
j=0

algo complicada, en todo caso.


6. Sistemas de ecuaciones

A veces, asociado a un cierto parametro, digamos n, tenemos dos (o mas) sucesiones de


n
umeros. Calcular un termino de una de estas sucesiones puede involucrar, tanto a terminos
de su propia sucesion, como a terminos de la otra (u otras). En estos casos, tendremos un
sistema de ecuaciones de recurrencia.
umero de listas de longitud n con ceros y unos con un
Ejemplo 6.1.16 Calculemos an , el n
n
umero par (o cero) de ceros.
Como es igualmente probable el que aparezca un n
umero par o un n
umero impar de ceros, la
intuici
on nos dice que an = 2n /2 = 2n1 . Pero veamos. Nos jamos en el smbolo que puede
aparecer en la u
ltima posici
on: si es un 1, el resto de la lista no es sino una de longitud n 1
con ceros y unos con un n
umero par de ceros (de las que hay an1 ). Pero si aparece un 0, lo
que queda es una lista de longitud n 1 con un n
umero impar de ceros.

Vaya!, mezclamos aqu otro tipo de ob1 an1





jetos. Inventemos un nombre para ellos y
n
umero par de ceros
veamos como salimos del aprieto. Si lla- an
0 bn1

umero de n-listas de ceros


mamos bn , al n



n
umero impar de ceros
y unos con un n
umero impar de ceros, podremos establecer la biyeccion que aparece a la derecha, que nos dice que, para n 1,
an = an1 + bn1 ,
on)
siendo los valores iniciales a1 = b1 = 1. Pero como hemos introducido una nueva (sucesi
inc
ognita, se hace necesario establecer una ecuacion para ella. Con un razonamiento an
alogo al anterior, pero ahora para las listas que cuenta bn , llegamos al siguiente sistema de
ecuaciones de recurrencia,

 



an
1 1
an1
an = an1 + bn1
o bien
=
.
()
bn = an1 + bn1 .
bn
bn1
1 1
A la derecha hemos escrito el sistema en forma matricial; como veremos en la subseccion 6.2.3,

las tecnicas del Algebra


Lineal, como el c
alculo de autovalores y la diagonalizaci
on, son las
herramientas adecuadas para la resoluci
on del problema. Observese, de todas formas, que este
ejemplo tan sencillo se puede resolver a mano. De () deducimos que an = bn . Ademas,
umero total de listas, 2n . De donde obtenemos que an = 2n1 para cada n.
an + bn es el n
(versi
on preliminar 23 de octubre de 2008)

6.1. Algunos ejemplos

357

7. Desdoblamiento
En ocasiones, una sucesion de n
umeros obedece a varias ecuaciones de recurrencia; o, mas
bien, en la sucesion conviven varias subsucesiones (por ejemplo, los terminos de ndice par y
los de ndice impar), cada una de las cuales tiene su propia ecuaci
on.
 1
n
x

, para cada entero n 0.


Ejemplo 6.1.17 Queremos calcular la integral Jn =
1 x2
0
Para calcular J0 usamos el cambio de variables x = sen(t):

J0 =

1
dx =
1 x2

/2

cos(t)
dt = .
cos(t)
2

un m
as sencilla:
La integral J1 es a
 1
1

x

2

dx = 1 x  = 1 .
J1 =
2
0
1x
0
La relacion de recurrencia se obtiene integrando por partes, como en el ejemplo 6.1.4: elegimos

du = (n 1) xn2 ;
u = xn1 ,

x
dx ,
v = 1 x2 ,
dv =
2
1x
para obtener que, para n 2,
Jn = x

n1

1

x2 
0


+ (n 1)

n2


1

x2 dx

= (n 1)

xn2

1 x2 dx .

Parece que nos hemos salido de la familia de integrales Jk ; pero un sencillo truco nos permite
volver a ella:
 1
 1

1 x2
n2
2
x
1 x dx = (n 1)
xn2
dx
Jn = (n 1)
1 x2
0
0
 1
 1
xn2
xn

dx (n 1)
dx .
= (n 1)
1 x2
1 x2
0
0






Jn2

Jn

Reordenando terminos llegamos a que


Jn =

n1
Jn2 ,
n

para cada n 2.

Esta recurrencia es f
acil de resolver, basta iterarla. Observemos que los terminos pares y los
impares forman dos sucesiones independientes. Por ejemplo, en el caso par, n = 2k,
J2k =

2k 1
(2k 1)(2k 3)
(2k 1)(2k 3) 3 1
J2k2 =
J2k4 = =
J0 .
2k
(2k)(2k 2)
(2k)(2k 2) 4 2 
=/2

(versi
on preliminar 23 de octubre de 2008)

358

Captulo 6. Ecuaciones de recurrencia

Multiplicando numerador y denominador por los factores pares que faltan, llegamos a que
 

(2k)!
(2k)!
2k

(2k)!
.
J2k =
2 2 =
2 2 =
2 2 =
2k+1
k
k
k
2
(2k (2k 2) 4 2)
(2 k (k 1) 2 1)
(2 k!)
Una manipulaci
on parecida llev
o a Wallis a una f
ormula para estimar el valor de (vease la
subseccion 2.4.4). Y en el caso impar, n = 2k + 1,
J2k+1 =

2k
(2k)(2k 2)
(2k)(2k 2) 4 2
J2k1 =
J2k3 = =
J1 .
2k + 1
(2k + 1)(2k 1)
(2k + 1)(2k 1) 5 3 
=1

Reescribiendo adecuadamente los factores, obtenemos que


 k 2
2 k!
22k
1
 .
=
J2k+1 =
(2k + 1)!
2k + 1 2k
k
Observese que se tiene que
1

,
2 2k + 1
de manera que bastara calcular una de las dos sucesiones.
J2k J2k+1 =

8. Modelos din
amicos discretos
Es frecuente que el an
alisis de como evoluciona una cierta cantidad en el tiempo nos
permita especicar la tasa de variaci
on por unidad de tiempo. Esta informaci
on da lugar,
de manera natural, a una ecuaci
on de recurrencia, como ilustramos en los siguientes dos
ejemplos.
Ejemplo 6.1.18 Sobre tipos de interes y prestamos.
Sabe el lector realmente como se devuelve un prestamo o como se amortiza una hipoteca?
En la escritura de un prestamo hipotecario, all
a por las u
ltimas p
aginas, aparece una f
ormula
algo misteriosa que determina, en funci
on de los par
ametros del contrato (vencimiento, tipo
de interes, etc.), el pago mensual al que nos comprometemos. De donde sale esa formula?
Por si el lector no esta ducho en la materia, empezaremos con una peque
na introducci
on a
las nociones de tipo de interes y capitalizacion.
El inter
es es la cantidad que se percibe por un prestamo de dinero en un periodo de
tiempo. Tras ese periodo de tiempo, una unidad de dinero, digamos un euro, se transforma en
1

1+I

(capital + intereses).

Si la cantidad inicial es de, por ejemplo, M euros, tras el periodo de tiempo, se tendran
M (1 + I) euros. La cantidad I es el tipo de inter
es, el interes percibido por un prestamo
10
de una unidad de dinero .
10

Es conveniente se
nalar que el tipo de interes que consideraremos aqu est
a expresado en tantos por uno. Es
frecuente que un tipo de interes se exprese tambien en tantos por ciento, as que habr
a que hacer la conversi
on
correspondiente. Por ejemplo, un tipo de interes I = 0,1 en tantos por 1 es lo mismo que uno del 10 %.

(versi
on preliminar 23 de octubre de 2008)

6.1. Algunos ejemplos

359

Un metodo de capitalizaci
on es un contrato que contiene unas reglas de acumulaci
on
de pagos de intereses sobre un capital dado. Para especicar un contrato de capitalizaci
on
(compuesta) se requieren los siguientes datos:
un periodo de tiempo t (normalmente, un a
no),
un tipo de interes R (en tantos por uno) asociado a ese periodo de tiempo;
y una frecuencia de capitalizaci
on, m (generalmente, un divisor de 12 o incluso m = ).
En un contrato como este decimos que capita0
t
lizamos cada t/m. Por ejemplo, podramos tener
1
2
3
m1
t m t m t
t
m
m
un contrato con un tipo de interes R anual y una
frecuencia de capitalizacion mensual; esto correspondera a tomar t = 1 a
no y m = 12. La
regla de capitalizaci
on asociada a los tres datos del contrato signica que
en tiempo 0, el capital es C.



R
En tiempo t/m, el capital acumulado sera C 1 + m
, el correspondiente al interes
producido por el tipo de interes (que esta asociado a t) durante ese periodo de tiempo.


R 2
En tiempo 2 t/m, el capital acumulado sera C 1 + m
, porque aplicamos la regla
de interes al capital acumulado hasta el vencimiento anterior.
As calcularamos el valor
en cada instante intermedio. En tiempo t, el
m
 delRcapital
, pues hemos efectuado m capitalizaciones sucesivas.
valor del capital sera C 1 + m
La capitalizaci
on podra ir m
as alla. Por ejemplo, el capital en tiempo 2 t = 2m (t)/m


R 2m
. En general, el valor en tiempo t, donde t es un m
ultiplo de t/m,
sera C 1 + m
t
,
t=k
m


sera C

R
1+
m

k
.

Amortizaci
on de un pr
estamo. Pasemos al problema que nos interesa, el analisis de
como se devuelve un prestamo. Un banco nos presta una cantidad V y tenemos N a
nos para
devolverlo. Por supuesto, tendremos que pagar unos intereses, que siguen una regla de una
composicion mensual con tipo de interes anual R. Es decir, cada mes pagaremos un interes
(que calcularemos con R) por la cantidad de dinero que debamos en ese momento al banco;
y ademas querremos ir devolviendo el capital prestado.
El metodo mas habitual en los prestamos hipotecarios es el llamado sistema franc
es,
en el que se paga una cantidad ja P cada mes. Con el resto de los datos del contrato,
queremos calcular cu
al es el montante de ese pago mensual jo. Llamemos Dn a la deuda
que mantenemos con el banco tras n meses:
En el instante inicial, debemos la cantidad total, D0 = V .


R
Tras el pago del primer mes, D1 = V 1 + 12
P , porque se han acumulado unos
intereses y hemos realizado un pago de P .


R
Tras el segundo mes, D2 = D1 1 + 12
P ; ahora acumulamos intereses sobre la deuda
que mantenamos con el banco.
(versi
on preliminar 23 de octubre de 2008)

360

Captulo 6. Ecuaciones de recurrencia


En un mes cualquiera, la deuda con el banco ser
a el resultado de restar a la deuda del
mes anterior (mas los intereses correspondientes) el pago jo de P . Esto es,


R
P
para cada n 1.
Dn = Dn1 1 +
12
Al terminar el contrato D12 N = 0.

Llamemos, por comodidad, a = 1 + R/12. Queremos resolver


Dn = a Dn1 P

para n 1 ,

junto con la condici


on inicial D0 = V .

a obtener
Disponemos ademas de la condici
on extra D12 N = 0, que como veremos nos permitir
el valor de P . De nuevo resolvemos la ecuacion de recurrencia por iteraci
on:
Dn = a Dn1 P = a (a Dn2 P ) P = a2 Dn2 aP P = a3 Dn3 a2 P aP P
an 1
,
= = an D0 P (an1 + an2 + + a2 + a1 + 1) = an D0 P
a1
donde hemos utilizado la f
ormula de la suma de una progresi
on geometrica nita. Como
D0 = V y a = 1 + R/12, tras unas cuantas manipulaciones llegamos a que



R n
12
12
P + 1+
P .
V
Dn =
R
12
R
a obtener que el valor de P es
Utilizando que D12 N = 0, el lector podr


1
R
.
P =V
12 1 (1 + R/12)12 N
En cada mes, parte del pago P se destina al pago de los intereses, y otra parte a la amortizacion de capital. Estos repartos varan durante la vida del prestamo. Sea In a la cantidad
invertida en intereses y Cn la destinada a capital, de manera que P = In + Cn . En el mes n
se pagan los intereses generados desde el u
ltimo pago, esto es, In = Dn1 R/12.
Animamos al lector a que obtenga las expresiones explci60000
tas de In y Cn a partir de las de Dn y P . Si, por ejemplo, el
50000
montante del prestamo es de V = 60000 euros, se devuelve
40000
en n = 20 a
nos y el tipo de interes anual es R = 0,05, el pago
30000
mensual (jo) resulta ser P = 395, 97 euros. Las gr
acas de
20000
la
izquierda
muestran
c
o
mo
va
disminuyendo
la
deuda
con
10000
el banco a lo largo de los 240 meses de vida del prestamo
0
50
100
150
200
(gr
aca superior) y que parte del pago mensual se destina
400
a intereses y que parte a capital (gr
aca inferior; como es
bien sabido, al principio pagamos sobre todo intereses) El
300
lector podr
a extraer conclusiones sobre la conveniencia de
200
esa practica, tan habitual, de cancelar el prestamo cuando
esta pr
oximo al vencimiento. Existen otros sistemas de amor100
tizacion de prestamos, que explicamos en los ejercicios 6.1.4
0
y 6.1.5.

50
100
150
200
(versi
on preliminar 23 de octubre de 2008)

6.1. Algunos ejemplos

361

Ejemplo 6.1.19 C
omo se administran medicamentos.
Para tratar una cierta enfermedad tomamos una cantidad ja C de medicamento cada N
horas. Buscamos un modelo matematico que permita determinar los valores de los parametros
C y N para que (1) la cantidad de medicamento en el organismo no supere nunca un cierto
umbral de toxicidad B; y (2) que esa cantidad de medicamento siempre este por encima de
un umbral de ecacia A, por debajo del cual el medicamento no surte efecto.
Tras cada N horas, s
olo una fracci
on d (un n
umero entre 0 y 1) de la cantidad de medicamento presente en el organismo se mantiene; el resto desaparece. Digamos que esta eliminacion sigue un modelo de decaimiento exponencial: si en un cierto momento hay una cantidad
D de medicamento, tras un tiempo t solo quedar
a D ek t , donde k > 0 es una cantidad (la
constante de decaimiento) que depender
a de diversos factores y que supondremos conocida.
Esto supone que, en realidad, d no es un nuevo par
ametro, sino que esta relacionado con N
mediante d = ekN . Si N muy grande, es decir, las tomas estan muy espaciadas, entonces d
sera muy cercano a 0, y casi todo el medicamento desaparecera entre una toma y la siguiente. Vamos a suponer, por simplicidad, que el paso del medicamento al torrente sanguneo
es instantaneo. Las tomas se producen en tiempos t0 , t1 , t2 , . . . , separadas cada N horas.
Llamaremos yn e xn , respectivamente, a las cantidades de farmaco en sangre justo antes e
inmediatamente despues de la toma de tiempo tn .
Los n
umeros yn cumplen que
yn = d xn1

para cada n 1,

porque s
olo se mantendra una fracci
on d de la cantidad presente tras la toma anterior.
Asimismo, se cumple que xn = yn + C para cada n 1 y, por tanto,
xn = d xn1 + C ,
ecuacion que se resuelve por simple iteracion (y utilizando que x0 = C). Dejamos al lector la
comprobaci
on de que las f
ormulas correspondientes son
1 dn+1
para cada n 1.
1d
Nuestro objetivo es elegir valores de los par
aB
metros C (cantidad en cada toma) y de N
x4
x3
x2
(periodo de tiempo entre tomas; recuerdese
x1
que jar N equivale a especicar d) de max0 = C
nera que, como se muestra en el dibujo de la
y4
y3
izquierda, la cantidad de medicamento en sany2
y1
gre se mantenga siempre entre los dos lmites
A
A y B (por supuesto, C ha de estar ya en el
t0
t1
t2
t3
t4
rango adecuado).
El asunto es m
as delicado de lo que pudiera parecer. Observemos que ambas sucesiones,
(xn ) e (yn ), son crecientes, pues d < 1, y necesitamos que, para cada n,
yn = C

d dn+1
1d

xn = C

1 dn+1
B
1d

xn = C

y que

yn = C

d dn+1
A.
1d

(versi
on preliminar 23 de octubre de 2008)

362

Captulo 6. Ecuaciones de recurrencia


C

La condici
on de la derecha se cumplir
a si el primero de
los yn , esto es, y1 = dC, lo hace. Para la de la izquierda,
sera suciente con exigir que se cumpla para el
ultimo de
los xn , esto es, para el lmn xn , que vale C/(1d). As que
debe cumplirse que

A
C
-d
B y dC A =
C B(1 d) .
0
1
1d
d
C
Y esto no es siempre posible. La region del plano (d, C) de6
terminada por la doble desigualdad anterior es la encerrada
entre las gr
acas de A/d y B(1d). . . y esa regi
on bien pudieB
ra ser vaca! En los dibujos observamos las dos posibilidades:
si B esta cercano a A, el problema no tiene soluci
on.
A
Un an
alisis mas detallado, que dejamos como ejercicio 6.1.6
-d
al lector, nos dice que tendremos toda una regi
on de pares de
1
0
nar la
valores (d, C) admisibles para el sistema siempre que11 B 4A. Lo que nos permite dise
estrategia de tomas (decidiendo el tiempo entre tomas o jando la cantidad de medicamento
por toma). El ejercicio 6.1.6 tambien contiene estos detalles.

9. Una estrategia de supervivencia


Cerramos esta seccion con un ejemplo, algo inclasicable, en el que aparecen muchas de
las tecnicas y argumentos que hemos visto hasta aqu.
Ejemplo 6.1.20 El problema de Josefo.
El que vamos a explicar a continuaci
on es, quiz
as, uno de los primeros problemas combinatorios de la Historia, una variaci
on sobre el
n
2
problema original que preocupaba a Flavio Josefo12 . Tenemos n pern1
3
sonas sentadas a una mesa circular y vamos eliminando a la segunda
persona que nos vayamos encontrando en el sentido de las agujas del
4
reloj (empezando a contar desde la primera posicion), hasta que s
olo
on iniuna sobreviva. El desafo, por supuesto, es calcular la posici
cial, que llamaremos J(n), que debe ocupar una persona si quiere sobrevivir. Instamos al
lector inquieto a que se entrene calculando a mano los primeros casos:
1

10

11

12

13

14

15

16

J(n)

11

13

15

11
La ratio entre B y A depende del medicamento en cuesti
on, y en los casos reales puede llegar a ser de
tan solo 2 a 1 (aqu estamos exigiendo 4 a 1). Aunque esta raz
on se puede reducir si nos damos un grado de
libertad extra, como es el que la primera toma sea especial (y algo mayor, por ejemplo).
12
El historiador judo Flavio Josefo (37-circa 100) particip
o en la revuelta contra Roma del a
no 66 y escap
oa
la matanza que tuvo lugar despues de la toma de la ciudadela de Josapata. Se cuenta, quiz
as ya con un tono
algo legendario, que 41 rebeldes fueron cercados por las tropas romanas y que, antes de ser capturados, optaron
por el suicidio colectivo. S
olo Josefo y otro compa
nero no parecan muy convencidos de la utilidad del sacricio,
as que nuestro personaje propuso el siguiente sistema de inmolaci
on: sentados a una mesa circular, se ira
eliminando a cada tercera persona hasta que s
olo dos sobrevivieran. Estos dos u
ltimos deberan ser los u
ltimos
en morir. Josefo calcul
o las posiciones que deban ocupar el y un compa
nero para sobrevivir al proceso.

(versi
on preliminar 23 de octubre de 2008)

6.1. Algunos ejemplos

363

Observese que todas las posiciones de supervivencia son impares (como corresponde a
que en la primera pasada eliminamos todas las posiciones pares). La tabla parece sugerir un
cierto patr
on (de bloques di
adicos) en esas posiciones de supervivencia.
1
1
Empezamos con el caso par, con n = 2k per2k1
2
3
2k
sonas. Queremos evaluar J(2k). En la pri2k3
2k1
5 mera pasada eliminamos a todas las perso3
nas que ocupen posiciones pares, como se
4
7
muestra en la gura. Queda una conguracion con k personas, aunque ahora no numeradas de 1 a k. Si estuvieran bien numeradas,
la posicion de supervivencia vendra dada directamente por J(k). Observemos que la posicion 2 ahora est
a etiquetada con 3, la 3 con 5, la 4 con 7, la 5 con 9, etc. Esto es, hemos
doblado cada posicion y le hemos restado 1. Si m es la posicion superviviente con el orden (1, 2, . . . , k) (de manera que m = J(k)), con el nuevo etiquetado (1, 3, 5, . . . , 2k 1) el
superviviente ser
a 2m 1. As llegamos a que
()

J(2k) = 2 J(k) 1 ,

para cada k 1.
1

El caso impar, n = 2k + 1, es analogo.


2k+1
2k+1
2
5
Ahora la primera pasada elimina a las per2k1
2k
7
3
sonas que ocupan posiciones pares, y a
un sobreviven k+1 personas. Observamos tambien
4
9
que la siguiente vctima es la que ocupaba la
alogos
posicion 1. Tras estos pasos, nos queda la conguraci
on de la gura. Con argumentos an
a los de antes (cuyos detalles dejamos al lector) establecemos que, para cada k 1,
()

J(2k + 1) = 2 J(k) + 1 .

Las reglas () y (), junto con el valor J(1) = 1, permiten generar la sucesi
on (J(n)). En
general, la simple iteraci
on de estas dos expresiones no permite hallar una f
ormula explcita
para J(n), pues en cada paso hay que comprobar la paridad de los sucesivos valores que
vamos encontrando. Aunque si, por ejemplo, n fuera una potencia de 2, digamos n = 2m ,
entonces el lector podra comprobar, por simple iteraci
on, que J(2m ) = 1.
Pero, y si escribimos n en binario, n = (ak , ak1 , . . . , a1 , a0 )2 , donde los aj {0, 1} y
on () requiere calcular el n
umero de Josefo en
ak = 1? Si n es par (esto es, si a0 = 0), la relaci
ltima cifra del
n/2, cuya expresion binaria es (ak , ak1 , . . . , a2 , a1 )2 (solo hay que eliminar la u
desarrollo binario). Por otro lado, si n es impar, () nos conduce a (n 1)/2, cuya expresion
en binario, como puede comprobar el lector, es tambien (ak , ak1 , . . . , a2 , a1 )2 .
En estos nuevos terminos, la relaci
on de recurrencia se simplica notablemente:
J ((ak , ak1 , . . . , a1 , a0 )2 ) = 2 J ((ak , ak1 , . . . , a1 )2 ) + (1)a0 +1 ,
pues sumamos 1 dependiendo del valor de a0 . Iterando esta regla y utilizando que ak = 1,
el lector podra llegar a que, si (ak , ak1 , . . . , a1 , a0 )2 es la expresion en binario de n, entonces
J ((ak , . . . , a0 )2 ) =

k


(1)aj +1 2j .

j=0

(versi
on preliminar 23 de octubre de 2008)

364

Captulo 6. Ecuaciones de recurrencia

6.1
EJERCICIOS DE LA SECCION
6.1.1 Este ejercicio hace referencia a la sucesi
on de n
umeros de Leibniz (L(n)) del ejemplo 6.1.14.
 
(a) Compruebese, recordando la discusi
on de la subsecci
on 4.2.4, que #{k n : nk es impar} = 2L(n) .
(b) Compruebese, por inducci
on, que si n 0, entonces

6.1.2 Sea In =

n
2
1

2L(k) = 3n .

k=0
1

xn ex dx, n 0.

(a) Compruebese que I0 = e 1 y obtengase que In = e nIn1 , para cada n 1.


(b) Para resolver la recurrencia anterior, consideramos los n
umeros Jn = (1)n+1 In /(n! e), para cada
n 0, que como podr
a comprobar el lector, verifican que
Jn = Jn1 +

(1)n+1
n!

para cada n 1.

Resuelvase esta recurrencia para obtener una f


ormula para Jn y la correspondiente f
ormula para In .

6.1.3 Definamos Hn =
et sinn (t)dt . Compruebese que H0 = 1 y H1 = 1/2, y que
0

Hn =

n(n 1)
Hn2 ,
1 + n2

para cada n 2.

Ded
uzcanse las f
ormulas para las integrales de ndice par H2k y las de ndice impar H2k+1 .
6.1.4 Este ejercicio y el siguiente utilizan los terminos del ejemplo 6.1.18. En un sistema alternativo
de amortizaci
on de prestamos, cada mes se paga una cantidad Pn , con la que se pagan los intereses
In generados en el periodo anterior y se amortiza una cantidad fija C de capital: Pn = In + C, para
n 1. Ahora el pago mensual no es fijo, sino que va decreciendo con el tiempo.
uzcase, de la condici
on
(a) Obtengase una f
ormula para Dn , la deuda con el banco en el mes n. Ded
D12 N = 0, el valor de C.
(b) Con estas expresiones, obtenganse los valores de In y de Pn . Dib
ujese la gr
afica de los pagos de
intereses y capital para los datos R = 0,05, N = 20 a
nos y V = 10 millones de euros.
6.1.5 Una tercera forma de amortizaci
on es el llamado sistema americano. Dados los datos R,
V y N como antes, se devuelve toda la deuda y sus intereses en un u
nico pago al final del contrato.
Esto es, se hace un pago en el mes 12 N de
Pfinal = V


12 N
R
1+
.
12

Para acumular ese capital se crea un fondo al que se contribuye con una cantidad fija P cada mes;
este dinero se capitaliza mensualmente con un tipo de interes anual S (generalmente, distinto de R).
on del mes n. Y, con la
Se pide establecer una f
ormula para An , el capital acumulado tras la aportaci
condici
on de que A12 N = Pfinal , obtener el valor de P .
6.1.6 Estamos con el modelo de toma de medicamentos explicado en el ejemplo 6.1.19.
(a) H
allense los rangos de los par
ametros d y C en los que mantendremos la cantidad de medicamento
entre los lmites A y B. Compruebese que si B 4A, estos rangos realmente existen.

(versi
on preliminar 23 de octubre de 2008)

6.1. Algunos ejemplos

365

(b) Supongamos que A = 3 y B = 16. Determnense los rangos para los par
ametros d y C.
(c) Supongamos que el medicamento est
a caracterizado por un valor de k = 0,1. Si C = 10, que rango
para N sera posible? y cu
al sera el rango admisible para C si quisieramos que N = 8 horas?
6.1.7 Estamos con el problema de Josefo del ejemplo 6.1.20. Supongamos que la expresi
on binaria
de n es (ak , ak1 , . . . , a1 , a0 )2 , donde ak = 1.
(a) Consideremos el n
umero N cuya expresi
on en binario tiene la misma longitud que la de n y sus
k
cifras son todo unos; esto es, N = j=0 2j . Calc
ulese 2n N y compruebese que coincide con J(n).
(b) Ded
uzcase de lo anterior que si escribimos n = 2k + l, con 0 l < 2k , entonces
J(2k + l) = 2l + 1 .
(c) Aplquese esta f
ormula al c
alculo de J(134) y J(1356).
(d) Ded
uzcase que
J((ak , ak1 , . . . a1 , a0 )2 ) = (ak1 , ak2 , . . . , a1 , a0 , ak )2 ,
de forma que podemos obtener J(n) sin m
as que permutar cclicamente la expresi
on binaria de n.
(e) Ciertos n
umeros de Josefo cumplen que J(n) = n/2. Es el caso, por ejemplo, de 2, 10, 42, 170,
etc. Caractercense los n
umeros para los que esto ocurre.
6.1.8 En el problema original de Josefo, se elimina a la tercera persona que vamos encontrando en
el sentido de las agujas del reloj. L
ancese sin miedo el lector a analizar esta cuesti
on para descubrir
que, si n = 41, entonces las dos posiciones que sobreviven son la 16 (pen
ultimo) y la 31 (
ultimo).
6.1.9 Consideremos una funci
on f : X X , donde X = {1, . . . , N } (f no tiene por que ser
necesariamente una biyecci
on, es decir, una permutaci
on de X ). Sea a0 X . Definimos a1 = f (a0 ),
a2 = f (a1 ) y, en general, an = f (an1 ). Compruebese que existe un n0 y un k tales que an+k = an
para todo n n0 .
6.1.10 Analcense, a la luz del ejercicio anterior, los siguientes sistemas din
amicos para diversos
valores de a0 :
(a) El conjunto X = {0, 1, . . . , 6} y la funci
on f : X X , donde f (n) es el resto de dividir 3n por 7.
(b) Sea X = {0, 1, . . . , 9999}. La regla f es la siguiente: dado n X , consideramos los n
umeros a (que
tiene las mismas cifras que n, pero ordenadas de menor a mayor) y b (mismas cifras que n, pero de
mayor a menor). Finalmente, f (n) = b a. Por ejemplo, si n = 3075, entonces a = 0357 y b = 7530,
de manera que f (n) = 7173.
6.1.11 Variamos el esquema de reproducci
on de Fibonacci del ejemplo 6.1.7 de la siguiente forma:
los conejos tienen dos parejas de descendientes cuando tienen 2 meses, y tres parejas a partir del
umero de conejos que en el mes n tienen 3 meses de vida o m
as,
tercer mes de vida. Llamemos an al n
bn a los que tienen exactamente 2 meses y cn a los de 1 mes. Suponemos que c0 = 1 y a0 = b0 = 0.
Escrbase una recurrencia (matricial) para (an , bn , cn ).
6.1.12 Considerese la sucesi
on (u(n))
n=1 de signos 1 dada por la siguiente regla:
u(2n) = u(n) ,

para cada n 1

u(2n + 1) = (1)n ,

para cada n 0.

(a) Escrbanse los primeros terminos.


(b) Observemos que todo n
umero natural n se puede escribir de manera u
nica en la forma n =
2r (2s + 1), con r 0 y s 0, sin m
as que agrupar los doses de su desarrollo en primos. Compruebese
que la regla de recurrencia anterior se puede escribir como sigue:
u(2r (2s + 1)) = (1)s .

(versi
on preliminar 23 de octubre de 2008)

366

6.2.

Captulo 6. Ecuaciones de recurrencia

Resoluci
on de ecuaciones de recurrencia

Como el lector podra imaginar, tras leer la secci


on anterior, la variedad de ecuaciones de
recurrencia que se pueden plantear es inmensa. En la mayora de las ocasiones, resolverlas es
una tarea difcil. En esta seccion nos vamos a limitar a estudiar algunos casos particulares,
sobre todo las ecuaciones lineales. Hay metodos para resolver otras familias de ecuaciones,
pero dado que las funciones generatrices, que introduciremos en el captulo 10, nos permitir
an
abordar estos problemas mas f
acilmente, obviaremos su descripcion.

6.2.1.

Ecuaciones lineales, homog


eneas y con coecientes constantes

En esta subseccion nos interesaremos por las sucesiones (an )


on de
n=0 que son soluci
ecuaciones como la siguiente:
()

A0 an + A1 an1 + + Ak ank = 0 para cada n k,

umeros dados, con A0 , Ak = 0, y k 1. Obserdonde los A0 , A1 , . . . , Ank son ciertos n


vemos que, de manera muy sintetica, estamos representando innitas condiciones para los
terminos de la sucesion. Sin perdida alguna de generalidad, podemos suponer (dividiendo ()
previamente por A0 ) que A0 = 1, as que reescribiremos la ecuacion de la siguiente forma:
()

an = B1 an1 + B2 an2 + + Bk ank

para cada n k,

para insistir en que cada termino de la sucesion (a partir de un cierto ndice) se escribe como
combinaci
on lineal (con coeficientes constantes) de unos cuantos terminos anteriores.
Observese como () es una receta para calcular el termino an si disponemos de los valores de
an1 , . . . , ank . La homogeneidad a la que nos referimos en el ttulo de la subseccion quiere
decir que no aparecen terminos extra que no dependan de los propios aj . Esto resulta ser
fundamental en el an
alisis que vamos a hacer. En la subsecci
on 6.2.2 veremos como podremos
tratar el caso en que aparezcan terminos no homogeneos.
El n
umero de terminos anteriores involucrados ser
a el grado de la ecuaci
on (la que
hemos escrito arriba sera de grado k). Necesitaremos, ademas, k condiciones iniciales para
que la sucesion arranque; en el caso de arriba, los n
umeros a0 , a1 , . . . , ak1 , los primeros k
terminos de la sucesion. A partir de ellos, y aplicando () reiteradamente, obtendremos
todos los valores de (an ). De todos los que hemos visto en la primera seccion de este captulo,
solo los ejemplos 6.1.1, 6.1.5, 6.1.6, 6.1.7 y 6.1.9 son de este tipo (aunque los ejemplos 6.1.2,
6.1.3, 6.1.18 y 6.1.19 conducan tambien a ecuaciones lineales con terminos no homogeneos,
que trataremos en la subsecci
on 6.2.2).
El caso k = 1, la ecuaci
on de primer grado, es especialmente sencillo, pues para
on
resolverla basta aplicar reiteradamente13 la regla hasta llegar al valor inicial. As, si la ecuaci
on es an = B1n a0
es an = B1 an1 , para cada n 1, y el valor inicial es a0 , entonces la soluci
para cada n 0 (veanse algunas variaciones en el ejercicio 6.2.1).
13

Como ya hemos hecho en varias ocasiones, incluso permitiendo la presencia de un termino extra constante
(ejemplos 6.1.3, 6.1.18 y 6.1.19). En realidad podemos resolver cualquier ecuaci
on del tipo xn = f (n)xn1 +
g(n), donde f (n) y g(n) son ciertas funciones, por simple iteraci
on. Aunque, como imaginar
a el lector, excepto
en algunos casos especialmente sencillos (vease el ejemplo 6.1.4), las f
ormulas que se obtienen son inmanejables.

(versi
on preliminar 23 de octubre de 2008)

6.2. Resolucion de ecuaciones de recurrencia

367

As que podemos restringir nuestro estudio al caso k 2. Y, de hecho, los ingredientes


del an
alisis general que vamos a desarrollar estan ya presentes en el caso de la ecuaci
on
lineal homog
enea de segundo orden, a la que a
nadimos dos condiciones iniciales:

an = an1 + an2
para cada n 2;
()
a0 = p , a1 = q .
Los datos son , , p y q, y el objetivo es obtener una f
ormula para an en terminos de n.
Aunque conviene recordar que no siempre la f
ormula cerrada para la soluci
on es la manera
mas ecaz de calcular los valores de la sucesion (la ecuacion de recurrencia podra ser un
mejor metodo). El interes por obtener una tal f
ormula est
a justicado, ademas de quizas por
razones esteticas, por que en muchas ocasiones nos permite entender el comportamiento
de los an (por ejemplo, para n grande) en terminos de ciertas cantidades que en un primer
vistazo podran pasar inadvertidas.
Una solucion de la ecuacion () es una sucesion (an )
erminos (a partir de a2 )
n=0 cuyos t
veriquen la ecuaci
on. La sucesion sera soluci
on del problema completo cuando, adem
as, los
dos primeros terminos, a0 y a1 , valgan p y q respectivamente. Observe el lector que la ecuacion
tiene, en principio, innitas soluciones (una por cada posible elecci
on de valores iniciales a0
y a1 ), pues basta arrancar la sucesi
on a partir de ese par de valores iniciales (por arrancar
la sucesi
on entenderemos jar dos valores iniciales para a0 y a1 y luego utilizar la ecuacion ()
olo una de estas sucesiones es, ademas,
para ir obteniendo sucesivamente a2 , a3 , a4 . . . ). Pero s
soluci
on del problema completo (justamente aquella para la que a0 = p y a1 = q)14 .
A. Estructura de las soluciones de la ecuaci
on
La estructura de las (innitas) soluciones de la ecuacion () es especialmente interesante.
Observese que ser solucion de la ecuaci
on nada dice sobre los valores que pudieran tener los
dos primeros terminos. Todo se debe a las siguientes dos propiedades:
on (cn ) denida mediante
1. Si (an ) y (bn ) son dos soluciones de (), entonces la sucesi
cn = an + bn para cada n 0 tambien lo es, puesto que, para cada n 2,
cn = an + bn = an1 + an2 + bn1 + bn2 = (an1 + bn1 ) + (an2 + bn2 )
= cn1 + cn2 .
umero real jo, entonces la
2. An
alogamente, si (an ) es solucion de () y r es un cierto n
sucesion (cn ) denida a traves de cn = r an para cada n tambien es solucion de ().
As que el conjunto de soluciones de () no es cualquier cosa, sino que tiene estructura, en
on lineal de sucesiones que sean solucion
concreto la de espacio vectorial15 : la combinaci
de () es una sucesion que sigue siendo solucion.
14
Animamos al lector interesado a que pruebe, utilizando el principio de inducci
on fuerte, que si dos suon de () y se cumple que a0 = b0 y a1 = b1 , entonces an = bn para todo n.
cesiones (an ) y (bn ) son soluci
Observese que los dos terminos que coinciden deben ser consecutivos para que funcione el argumento.
15
El lector avisado sabr
a identicar los conceptos de base y dimensi
on que aparecer
an implcitamente en
nuestros argumentos. Si adem
as est
a versado en la teora de resoluci
on de ecuaciones diferenciales, muchas de
las tecnicas que aparecer
an en esta secci
on le resultar
an familiares.

(versi
on preliminar 23 de octubre de 2008)

368

Captulo 6. Ecuaciones de recurrencia

Esta observaci
on tiene consecuencias muy relevantes. Digamos que (an ) y (bn ) son dos
soluciones distintas de la ecuaci
on (con mas precision, deben ser linealmente independientes,
lo que en este caso solo supone que no deben ser una m
ultiplo de la otra). Por simplicar
el argumento, supongamos que son (an ) = (0, 1, . . . ), la solucion que empieza con 0 y 1, y
(bn ) = (1, 0, . . . ), la que empieza con 1 y 0.
Ahora consideremos otra soluci
on cualquiera de la ecuaci
on, digamos (cn ) = (c0 , c1 , c2 , . . . ).
Formamos la sucesion (dn ) denida mediante
dn = c1 an + c0 bn

para cada n.

on lineal de soluciones.
Ya sabemos que (dn ) es solucion de la ecuacion, pues es una combinaci
Pero, adem
as, sus dos primeros terminos coinciden con los de (cn ). As que ambas sucesiones,
(cn ) y (dn ), han de ser la misma.
Si el lector se anima a rehacer el argumento anterior con dos sucesiones (an ) y (bn ) generales, como pedimos hacer en el ejercicio 6.2.2, llegara a la conclusi
on de que, si encontramos
dos soluciones distintas (linealmente independientes), cualquier otra soluci
on de la ecuacion
se puede escribir como combinaci
on lineal de ellas.
B. Forma de las soluciones de la ecuaci
on
Pues ya esta, no? Tomamos dos soluciones de la ecuacion, por ejemplo dos tan sencillas
como las que empiezan, respectivamente, por (0, 1, . . . ) y por (1, 0, . . . ). Aplicando reiteradamente la ecuaci
on, descubrimos que los primeros terminos de estas sucesiones son
(an ) 0 1 2 + 3 + 2
(bn ) 1 0

2 + 2

4 + 32 + 2
3 + 2 2

Y cualquier otra soluci


on de la ecuaci
on se escribira como combinacion lineal de estas dos:
su termino general sera de la forma
cn = Aan + Bcn ,
donde A y B son dos ciertas constantes (que en realidad quedaran determinadas en cuanto
jemos las condiciones iniciales).
Pero no, este no es el nal de la historia, porque no parece posible que de la expresi
on
anterior podamos extraer una f
ormula cerrada y manejable en funci
on del ndice n para el
termino general de una soluci
on cualquiera.
Ser
a, quizas, que no estamos eligiendo bien las dos soluciones especiales. Recordemos
que la u
nica restriccion es que han de ser sucesiones que veriquen la ecuacion y que sean
distintas (linealmente independientes). Pero parece razonable buscar soluciones con cierta
estructura.
Podramos, por ejemplo, buscar soluciones cuyos terminos esten en progresion aritmetica.
Fijamos entonces un primer termino a0 y exigimos que el siguiente sea de la forma a1 = a0 +d.
El par
ametro d esta a nuestra disposici
on. La ecuaci
on de recurrencia nos dice que el tercer
termino ha de ser a2 = (a0 + d) + a0 y buscamos un valor de d para el que ese termino sea
el siguiente en la progresion, esto es, a0 + 2d. Enseguida nos damos cuenta de que no vamos
por buen camino.
(versi
on preliminar 23 de octubre de 2008)

6.2. Resolucion de ecuaciones de recurrencia

369

Sin desanimarnos, lo volvemos a intentar, pero ahora con terminos que esten en progresi
on
geometrica. Empezamos, por ejemplo, con (1, r, . . . ). Los siguientes terminos son
(1, r, r + , (r + ) + r, . . . ) .
Ahora ajustamos r para que el tercer termino este en progresi
on geometrica con los anteriores;
esto es, r ha de cumplir que
r + = r2 .
Vamos con el cuarto termino. Querramos que ( r + ) + r = r 3 . Pero observese que
(r + ) + r = r 2 + r = r( r + ) = r 3 .
  
  
=r 2

=r 2

Oooh! (de asombro): sale gratis. El mismo valor de r que nos permite ajustar el tercer
termino consigue hacerlo con el cuarto. El lector podr
a comprobar que lo mismo ocurre para
los siguientes. As que existe una solucion de la ecuaci
on de la forma an = r n (recuerdese que
esto ya ocurra en las ecuaciones de primer grado), una f
ormula de lo m
as sencilla. Ademas,
la ecuaci
on de segundo grado que dene el valor de r adecuado puede tener otra soluci
on,
que nos permitira construir otra sucesi
on, tambien con terminos en progresi
on geometrica,
con la que completar el an
alisis del problema.
C. M
etodo de resoluci
on del problema completo
Ya estamos en condiciones de resumir y formalizar estas ideas. Buscamos soluciones (an )
de la ecuaci
on () cuyos terminos esten en progresion geometrica; es decir, que sean de la
umero no nulo que determinaremos
forma an = r n , para todo n 0, donde r es un cierto n
en un momento. El que sea soluci
on exige que
r n = r n1 + r n2

para cada n 2.

Pero estas innitas condiciones se resumen, en realidad, dividiendo por r n2 , en una sola,
r2 = r +
(el lector que haya estudiado el ejemplo 6.1.7 encontrar
a familiar este argumento). A la ecuacion algebraica as obtenida se le llama ecuaci
on caracterstica. El an
alisis depender
a de
que tipo de soluciones tenga esta ecuacion de segundo grado.
Caso 1. La ecuaci
on caracterstica tiene dos races reales distintas
Llamemos r1 y r2 a estas dos races. Sabemos que las sucesiones denidas por an = r1n
y bn = r2n son soluciones de (). Son adem
as, distintas (linealmente independientes), porque
on se escribir
a como combinacion lineal de ellas. Es
r1 = r2 . As que cualquier otra soluci
decir, podemos escribir la solucion general de () como
 n

A r1 + B r2n
donde A y B son dos constantes cualesquiera. Ahora buscamos la (
unica) soluci
on que verica
las dos condiciones iniciales, para lo que bastar
a determinar los valores de A y B adecuados
mediante sencillas relaciones algebraicas. Lo vemos en un ejemplo.
(versi
on preliminar 23 de octubre de 2008)

370

Captulo 6. Ecuaciones de recurrencia

Ejemplo 6.2.1 Consideremos la sucesi


on de n
umeros de Fibonacci (Fn ) dada por F0 = 0,
F1 = 1 y Fn = Fn1 + Fn2 para cada n 2.
2
La ecuacion caracter
stica correspondiente

es r r 1 = 0, cuyas soluciones son, como ya


on general de la ecuaci
on de
sabemos, r1 = (1 + 5)/2 y r2 = (1 5)/2; as que la soluci
Fibonacci es




1 5 n
1+ 5 n
+B
.
A
2
2
En esta f
ormula tenemos codicadas todas las soluciones de la ecuaci
on de Fibonacci. Queda
determinar los valores de A y B que corresponden a la sucesi
on que nos incumbe, la que tiene
como valores iniciales 0 y 1. Solo hay que escribir los casos n = 0 y n = 1 de la f
ormula y
resolver el sistema de ecuaciones correspondiente:

0 = F0 = A + B

1
1
 
 
y B = .
= A =
5
5
1 = F1 = A 1+ 5 + B 1 5
2

De manera que el n-esimo n


umero de Fibonacci se puede escribir como




1+ 5 n
1 5 n
1
1
Fn =

2
2
5
5
la llamada f
ormula de Binet, de 1843 (aunque Daniel Bernoulli y Euler ya la conocan
hacia 1724). Pasan cinco siglos desde que Fibonacci se interesara por estos n
umeros hasta
que se obtiene esta expresion. Nada menos que cinco siglos separan las simples manipulaciones
con la ecuaci
on del nivel de abstracci
on que supone el an
alisis que aqu hemos hecho y que
nos ha conducido a esta f
ormula.
F
ormula, por cierto, algo sorprendente; observese que los distintos terminos, que involucran races de 5, se combinan magicamente para dar siempre un entero. Quiz
as no sea la
as bien hace explcimanera mas ecaz de calcular el valor de un cierto Fn , perodescubre, o m
on aurea. Compruebe el
ta, la relaci
on entre estos n
umeros de Fibonacci y = 1+2 5 , la raz
lector que la f
ormula se puede reescribir como

 

1
1 n
n
.
Fn =

5
Ademas, como > 1,
el segundo sumando se hace muy peque
no cuando n es grande, de
manera que Fn n / 5 cuando n . Lo que demuestra el resultado que vimos en el
ejemplo 6.1.7 sobre el comportamiento asintotico de las razones entre n
umeros de Fibonacci
consecutivos:
Fn
=.
lm
n Fn1

M
as a
un, como 5 > 2,


n 
 1
 1
1


para cada n 0.
 5  < 2

oximo a n / 5. Si n es par, Fn estara ligeramente por encima


As queFn es el entero mas pr

de n / 5; y si n es impar, ligeramente por debajo.


(versi
on preliminar 23 de octubre de 2008)

6.2. Resolucion de ecuaciones de recurrencia

371

Caso 2. La ecuaci
on caracterstica tiene una raz real doble
nica raz (doble) r1 solo si los
La ecuaci
on caracterstica r 2 r = 0 tiene una u
coecientes y son muy especiales. En concreto, con la ayuda de la formula cuadr
atica (el
2
radicando + 4 ha de ser 0), obtenemos que tienen que ser de la forma = 2r1 y = r12 .
La ecuaci
on de recurrencia con la que estamos es, pues,
an = 2r1 an1 r12 an2 ,

para cada n 2.

on, pero falta otra. No


Sabemos que (an ) = (1, r1 , r12 , r13 , . . . ) es una solucion de la ecuaci
queremos que sea un m
ultiplo de esta, y parece razonable suponer que ha de depender de r1 .
Podramos intentarlo con la sucesi
on (bn ), que empieza con (0, r1 , . . . ) y que cumple las
condiciones requeridas. Miremos el aspecto de sus primeros terminos:
b2 = 2r1 b1 r12 b0 = 2r12 ,
b3 = 2r1 b2 r12 b1 = 4r13 r13 = 3r13 ,
b4 = 2r1 b3 r12 b2 = 6r14 2r14 = 4r14 .
De lo mas atractivo (y sorprendente): las sucesivas cancelaciones hacen que los terminos de la
ormula sencilla y manejable.
sucesion sean de la forma nr1n , para cada n 0, de nuevo una f
El lector puede comprobar m
as formalmente que, efectivamente, si r1 es una raz doble de la
on
ecuacion caracterstica, entonces la sucesion dada por bn = n r1n para cada n es una soluci
de la ecuaci
on de recurrencia. As que la soluci
on general se puede escribir como


A r1n + B n r1n
De nuevo, los valores de A y B se determinar
an con las condiciones iniciales.
Por supuesto, haba una raz
on, adem
as de las heursticas que hemos expuesto aqu, para
elegir la forma particular de esta segunda solucion (vease el ejercicio 6.2.3). No hay tal Deus
ex machina, al que a veces somos tan acionados en los textos de Matematicas.
Ejemplo 6.2.2 La ruina del jugador, revisada.
En el ejemplo 6.1.9 present
abamos los ingredientes del problema: empezamos a jugar con n
euros, donde 0 n N , para cierto N jo. En cada partida ganamos con probabilidad p y
perdemos con probabilidad q = 1 p. Si a(n) es la probabilidad de arruinarnos, entonces
a(n + 2) =

q
1
a(n + 1) a(n) ,
p
p

si 0 n N 2.

Las condiciones iniciales, un tanto inusuales, son a(0) = 1 y a(N ) = 0. La ecuaci


on
caracterstica es, en este caso,
q
1
r 2 r + = 0,
p
p
cuyas soluciones son r1 = q/p y r2 = 1. Llamemos, por comodidad, al cociente q/p. Hay
dos casos, dependiendo de si = 1 o no.
(versi
on preliminar 23 de octubre de 2008)

372

Captulo 6. Ecuaciones de recurrencia

Caso p = q (es decir, = 1). Observese que, en cada partida, la probabilidad de ganar
no es la misma que la de perder. Si estuvieramos hablando de una moneda, diramos que
esta cargada. Cuando jugamos a la ruleta, apostando al rojo o al negro, estamos en este
caso, como luego explicaremos (de hecho, en el caso en que la probabilidad de ganar p es
menor que la de perder q, como el lector se habra imaginado ya).
Como las races de la ecuacion caracterstica son distintas, la solucion general sera
a(n) = A 1n + B n .
Ahora, con ayuda de las condiciones iniciales, determinamos A y B:

a(0) = A + B = 1
1
N
y B=
,
= A =
N
N
1
1 N
a(N ) = A + B = 0
con lo que la soluci
on es
a(n) =

N
1
+
n ,
N
1
1 N

(donde, recordemos, = q/p).

Caso p = q = 1/2 Ahora hay una u


nica raz de la ecuaci
on caracterstica, r1 = r2 = 1, y la
soluci
on general se puede escribir como
a(n) = A 1n + B n 1n = A + Bn.
De las condiciones iniciales obtenemos que A = 1 y B = 1/N , con lo que
a(n) = 1

n
.
N

El comportamiento de las dos soluciones es bien diferente: en el caso p = q, es una funci


on
lineal de n, mientras que en el otro es una funci
on exponencial.
Para entender realmente que supone esto, supongamos que estamos en un casino europeo
apostando al rojo: de los 36 n
umeros, hay 18 rojos y otros 18 negros. Pero hay tambien una
casilla, la del 0, que no lleva color; si sale el 0, la banca se lleva todas las apuestas. Parece
poco negocio para el casino, pero. . . La probabilidad de ganar es p = 18/37, mientras que la
de perder es q = 19/37, ligeramente superior, debido a la presencia de esta casilla extra con
el 0. El cociente = q/p vale 1,0555. Pr
acticamente 1, pero no exactamente 1.
Las gracas que aparecen a continuaci
on comparan los valores de a(n) en el rango 0
n N con = 1 y = 1,0555. Marcamos en el eje horizontal los posibles valores de n,
nuestra fortuna inicial, y en el eje vertical los correspondientes de a(n), la probabilidad de
arruinarse cuando empezamos a jugar con n euros.
Con = 1, claro, tenemos una recta. Pero la gr
aca para = 1,0555 es bien diferente.
A la izquierda representamos el caso en que N = 40, en el que ya se aprecia una cierta
diferencia entre ambas gr
acas. La de la derecha corresponde a N = 200, y la discrepancia
es ya espectacular.
(versi
on preliminar 23 de octubre de 2008)

6.2. Resolucion de ecuaciones de recurrencia

373

0.8

0.8

0.6

0.6

0.4

0.4

0.2

0.2

10

20
n

30

40

20

40

60

80 100 120 140 160 180 200


n

Observese que, en la graca de la derecha, la probabilidad de arruinarse es pr


acticamente 1
(en el caso = 1, 0555) a menos que n este muy, pero que muy cercano a N = 200. Digamos
que estamos jugando a doblar o arruinarse. Es decir, partimos de una fortuna n que es la
mitad de N (N = 2n). Si el juego es equilibrado ( = 1), la probabilidad de arruinarse es
siempre un 1/2, sea cual sea N . Pero en la ruleta ( = 1,0555), para N = 40 tenemos una
probabilidad de ruina de casi el 70 %. . . y si estamos con N = 200, nos arruinaremos en m
as
del 98 % de las ocasiones. Las conclusiones quedan para el lector.

Caso 3. La ecuaci
on caracterstica tiene dos races complejas
La ecuacion caracterstica tiene coecientes reales, as que sabemos16 que las dos races
complejas son conjugadas una de la otra. Digamos que son z1 = a + bi y z2 = z 1 = a bi.
El lector puede comprobar que las sucesiones (an ) y (bn ) denidas, para cada n, mediante
an = z1n y bn = z2n respectivamente, son soluciones de la ecuacion de recurrencia, as que su
soluci
on general se puede escribir como
 

 n
A z1 + B z2n = A z1n + B z n1 ,
donde A y B, como siempre, se determinan con las condiciones iniciales.
Pero todo en este problema, los coecientes de la ecuacion y los propios terminos de
la sucesi
on, son n
umeros reales. Y obtenemos una solucion escrita en terminos de n
umeros
complejos. Pese a la advertencia de Hadamard (vease la p
agina 22), puede que esto no nos
deje muy satisfechos. . . y eso que, aceptando esta escritura con n
umeros complejos, la f
ormula
que se obtiene es de lo mas manejable!
As que intentaremos encontrar otro par de soluciones, que cumplan las condiciones habituales, cuyos terminos sean n
umeros reales y de manera que la formula que obtengamos siga
siendo razonable. Nuestras soluciones especiales empiezan con
(an ) = (1, z1 , . . . )

(bn ) = (1, z 1 , . . . ) .

Usando la forma polar, podemos escribir z1 = rei y z1 = rei . Pero entonces, recordando
la casi m
agica f
ormula de Euler-De Moivre, z1n = r n ein , mientras que z1n = r n ein . As que,
para cualquier n, z1n y z1n son uno el conjugado complejo del otro. Por lo tanto, z1n + z1n es
umero imaginario puro.
un n
umero real, mientras que z1n z1n es un n
16
En el caso de la ecuaci
on de segundo grado es obvio. Para ecuaciones de mayor grado, ya vimos el
argumento en la demostraci
on de la proposici
on 4.48.

(versi
on preliminar 23 de octubre de 2008)

374

Captulo 6. Ecuaciones de recurrencia


Esto nos sugiere denir dos nuevas sucesiones (
an ) y (bn ) mediante

an =

an + bn
2

an bn

bn =
2i

(para cada n 0)

cuyos primeros terminos resultan ser



1 + 1 z + z
1
1
,
, . . . = (1, Re(z1 ), . . . )

an =
2
2

1 1 z z
1
1
bn =
,
, . . . = (0, Im(z1 ), . . . )
2i
2i
Pero mas a
un, los terminos generales se pueden escribir de manera muy sencilla:

an =
bn =

z1n + z n1
ein + ein
= rn
= r n cos(n) ,
2
2
z1n z n1
ein ein
= rn
= r n sin(n) .
2i
2i

Perfecto: todos los terminos son reales, y la formula es sencilla. Decidimos entonces emplear,
para describir la soluci
on general de la ecuaci
on, la expresi
on


C r n cos(n) + D r n sen(n)
n=0

donde, recordemos, r = |z1 | y = arg(z1 ) y z1 es una de las races complejas de la ecuacion


caracterstica.
Ejemplo 6.2.3 Queremos encontrar la sucesi
on de n
umeros (an ) definidos por
an = 2 an1 2 an2 ,

para cada n 2,

junto con las condiciones iniciales a0 = 1 y a1 = 1.


De la ecuaci
on caracterstica r 2 2r + 2 = 0 obtenemos races

y
z2 = z 1 = 1 i = 2 ei 4 .
z1 = 1 + i = 2 ei 4
La soluci
on general de la ecuaci
on se puede escribir como


.
A (1 + i)n + B (1 i)n
n=0

Pero tambien con la siguiente expresion alternativa:


  

 

n + D sen
n
.
2n/2 C cos
4
4
n=0
Con las condiciones iniciales determinamos que C = 1 y que D = 0, as que la respuesta
buscada es, para cada n 0,
 
n .
an = 2n/2 cos
4
Compruebese que, a pesar de escribirse de esta forma algo extravagante, los terminos de la
sucesion son todos enteros.

(versi
on preliminar 23 de octubre de 2008)

6.2. Resolucion de ecuaciones de recurrencia

375

Todos los argumentos que hemos desarrollado hasta aqu para el caso de la ecuacion de
segundo grado se aplican, mutatis mutandis, a las ecuaciones lineales, homogeneas y con
coecientes constantes de grado k. Dejamos al lector que reconstruya la teora para el caso
general, y nos limitamos a exponer el resultado basico, as como a ilustrarlo con un ejemplo.
Teorema 6.1 Dado k 1, la soluci
on general de la ecuaci
on
an = 1 an1 + 2 an2 + + k ank
se puede escribir de la forma


n
n
n
,
P1 (n) r1 + P2 (n) r2 + + Ps (n) rs
n=0

on caracterstica
donde r1 , r2 , . . . , rs son las races distintas (reales o complejas) de la ecuaci
r k = 1 r k1 + 2 r k2 + + k1 r + k
y cada Pi (n) es un polinomio generico (con coeficientes arbitrarios) de grado igual a la multiplicidad de la correspondiente raz ri menos uno.
Ejemplo 6.2.4 Consideremos la ecuaci
on an = 4an1 6an2 +6an3 5an4 +2an5 , para
cada n 5, junto con las condiciones iniciales a(0) = a(1) = a(2) = a(3) = 0 y a(4) = 1.
Las races de la ecuacion caracterstica (una quntica)
r 5 = 4r 4 6r 3 + 6r 2 5r + 2 ,
son los n
umeros 2, 1 (raz doble), i y i (dos races complejas conjugadas, como debe ser).
As que, conforme al teorema anterior, la soluci
on general de la ecuacion de recurrencia se
puede escribir como


.
(A1 + A2 n)1n + A3 2n + A4 in + A5 (i)n
n=0

O, si queremos evitar la presencia de n


umeros complejos, como
 
 

n + B5 1n sin
n
.
(B1 + B2 n)1n + B3 2n + B4 1n cos
2
2
n=0
Si ahora imponemos las cinco condiciones iniciales, obtendremos un sistema de cinco ecuaciones de las que podemos determinar los valores de los n
umeros B1 , . . . , B5 . Dejamos al lector
la comprobaci
on (algo tediosa) de que la soluci
on del problema es la sucesion dada por


1
1
1
1
i [(i)n in ]
para cada n 0,
an = 2n n + +
5
2
10 20
o bien

 
 
1
1
1
1
n +
sin
n .
an = n + 2n cos
2
5
5
2
10
2
Ambas formulas, pese a su alambicada escritura, representan a la sucesion de n
umeros (enteros) cuyos primeros terminos son
(0, 0, 0, 0, 1, 4, 10, 22, 47, 98, 200, 404, 813, 1632, 3270, . . . )
El lector podr
a deducir, a partir de la f
ormula anterior, que an 2n /5 cuando n .
(versi
on preliminar 23 de octubre de 2008)

376

Captulo 6. Ecuaciones de recurrencia

6.2.2.

Ecuaciones lineales, no homog


eneas y con coecientes constantes

Consideramos ahora un caso m


as general, en el que la ecuaci
on contiene un termino extra
que depende del ndice n:
an + B1 an1 + B2 an2 + + Bk ank = f (n) ,
donde f (n) es una cierta funci
on no nula. De nuevo, y por simplicar la exposici
on, supongamos que nuestra ecuaci
on es de grado dos,
()

an + an1 + an2 = f (n)

para cada n 2.

Tendremos, ademas, dos condiciones iniciales, los valores de a0 y a1 .


Centremonos en las soluciones de la ecuacion (), a la que nos referiremos como ecuaci
on
inhomog
enea. Sean (an ) y (bn ) dos soluciones de () y consideremos la sucesion suma (cn )
denida por cn = an + bn para cada n. Observese que
cn + cn1 + cn2 = (an + bn ) + (an1 + bn1 ) + (an2 + bn2 )
 


= an + an1 + an2 + bn + bn1 + bn2 = f (n) + f (n) = 2f (n) .
No obtenemos f (n), as que (cn ) ya no es solucion de () y, por tanto, no tenemos la estupenda
estructura (vectorial) de antes.
Pero no todo est
a perdido. Al menos sigue ocurriendo que la soluci
on del problema (ecuacion inhomogenea mas valores iniciales) existe (basta arrancar la sucesion) y es u
nica. De
nuevo, los dos valores iniciales determinan completamente la solucion que nos interesa. As que
lo que buscamos es una forma adecuada de escribir la soluci
on general de la ecuaci
on (),
para lo que nos apoyaremos en lo que ya sabemos de las ecuaciones homogeneas.
Planteamos la ecuaci
on homog
enea asociada a nuestro problema:
()

an + an1 + an2 = 0 ,

para cada n 2.

De esta ecuacion sabemos como calcular su solucion general, que ser


a de la forma (Abn +Bcn ),
donde (bn ) y (cn ) son sucesiones solucion de () (con el aspecto que corresponda a cada caso,
dependiendo de las races de la ecuacion caracterstica).
Ahora viene la observaci
on clave: consideremos una solucion particular de la ecuaci
on ().
Vale una cualquiera, por ejemplo la que obtenemos arrancando la sucesi
on a partir de dos
valores iniciales que jemos. Llamemos (dn ) a esta solucion. Lo que armamos es que cualquier
sucesion de la forma


Abn + Bcn + dn ,
donde A y B son par
ametros, es solucion de la ecuaci
on inhomogenea (). Dejamos que el
lector haga la comprobaci
on pertinente.
Pero ademas, toda soluci
on de () se puede escribir como arriba. Es, una vez mas, un
argumento de unicidad: si (n ) es una solucion que empieza por (0 , 1 , . . . ), podemos encontrar A y B que determinen una sucesion que es solucion de () y que empieza como (n ).
As que han de ser la misma. Los detalles quedan como ejercicio 6.2.4 para el lector.
(versi
on preliminar 23 de octubre de 2008)

6.2. Resolucion de ecuaciones de recurrencia

377

El problema queda resuelto (excepto por un peque


no detalle que explicaremos luego). Si
on
queremos encontrar la expresi
on de la sucesion de n
umeros (an ) que cumplen la ecuaci
an + an1 + an2 = f (n) ,

para cada n 2,

junto con las condiciones iniciales a0 = p y a1 = q,


(1) analizamos primero la ecuacion homogenea asociada,
an + an1 + an2 = 0,
y determinamos la soluci
on general de esta ecuacion, que ser
a de la forma (Abn +Bcn ).
(2) Calculamos una soluci
on particular de la ecuacion completa; digamos que es la sucesion (dn ).
(3) Ya tenemos la solucion general de la ecuaci
on completa, (Abn + Bcn + dn ). Y ahora (y
no antes) imponemos las condiciones iniciales para determinar los n
umeros A y B.
on de
Ejemplo 6.2.5 Encontremos la sucesi
on de n
umeros (an ) que satisface la ecuaci
recurrencia an = an1 + an2 + 7, junto con las condiciones iniciales a0 = 0 y a1 = 1.
La ecuacion homogenea asociada es la de Fibonacci, de la que conocemos bien la forma de
la soluci
on (vease el ejemplo 6.2.1). Ahora nos damos cuenta (!) de que la sucesion cuyos
terminos son todos iguales a 7 es solucion de la ecuacion. N
otese que nos referimos u
nicamente a la ecuaci
on (los dos primeros valores de esta sucesion son, claro, 7 y 7). As que
la soluci
on general de la ecuaci
on completa se puede escribir como




1 5 n
1+ 5 n
+B
7.
A
2
2
alculos, determinamos
Imponiendo los valores iniciales a0 = 1 y a1 = 1, y tras unos laboriosos c
los valores de A y B y concluimos que la soluci
on que buscamos viene dada por

n 



1+ 5
1 5 n
7 9 5
7 9 5
+

+
7
para cada n 0.
an =

2
10
2
2
10
2
M
etodo de los coeficientes indeterminados
El detalle al que nos referamos es nuestra constante preocupacion por que la f
ormula
obtenida sea manejable. Desde luego, la expresi
on de la parte homogenea de la soluci
on lo
es, que para eso hicimos todo el esfuerzo antes. Pero, como obtener una soluci
on particular
con una expresi
on sencilla? Si, por ejemplo, la obtenemos partiendo de dos valores iniciales
cualesquiera, es casi seguro que no la tendr
a. Aunque no existe un metodo general para
obtener estas soluciones agradables, disponemos sin embargo de reglas (casi trucos) ad hoc
que se aplican cuando la funci
on f (n) tiene una forma especca. Queda siempre tambien,
claro, el metodo de prueba y error.
Lo vemos en un ejemplo, en el que tambien advertiremos las posibles dicultades del
procedimiento, que se denomina m
etodo de los coeficientes indeterminados. Se trata,
esencialmente, de buscar soluciones particulares dentro de la misma familia de funciones a
la que pertenezca f (n) (observese que, en el ejemplo anterior, la solucion particular era una
constante, como el termino no homogeneo).
(versi
on preliminar 23 de octubre de 2008)

378

Captulo 6. Ecuaciones de recurrencia


Digamos que nos enfrentamos con la ecuacion
an an1 an2 = 3n .

Buscamos una solucion particular de la ecuaci


on, e intentamos encontrar una de la forma
n
on, y que determinamos con

an = C3 , donde C es una constante que esta a nuestra disposici
la propia ecuaci
on:
an1 
an2 = 3n

an 

C3n C3n1 C3n2 = 3n

de donde obtenemos que C = 9/5. Y, efectivamente, 


an =
la ecuaci
on, como puede comprobar el lector.
Pero ahora imaginemos que la ecuacion es

9 n
53

9C 3C C = 9 ,

es una soluci
on particular de

an an1 an2 = n ,
donde es la razon aurea. Probamos, como antes, con una solucion del tipo 
an = C n :
an1 
an2 = n = C n C n1 C n2 = n = C( 2 1) = 2 .

an 
Y no hay manera de determinar C, porque 2 1 = 0. El metodo no funciona, no puede
funcionar, porque n es ya solucion de la ecuacion homogenea ( era una de las races de la
ecuaci
on caracterstica), as que no puede ser soluci
on de la completa.
Inspir
andonos en alg
un argumento que hicimos anteriormente, podramos intentar con
soluciones de la forma 
an = C n n :
an1 
an2 = n = Cn n C(n 1) n1 C(n 2) n2 = n

an 

5+ 5
2
=
.
= Cn( 1) + C(2 + ) = = C =
2+
10
Ya tendramos determinado el valor de C y, por tanto, una soluci
on particular.
Las ideas de este metodo se pueden aplicar tambien a ecuaciones en las que el termino no
homogeneo sea un polinomio o una funci
on seno o coseno. Como las funciones generatrices nos
permitir
an resolver todos estos casos (vease la seccion 10.4), no entraremos en m
as detalles.
2

6.2.3.

Sistemas de ecuaciones de recurrencia

Consideremos un sistema de dos ecuaciones de recurrencia lineales17 (de grado uno),


homogeneas y con coecientes constantes:

an = 1,1 an1 + 1,2 bn1 ,
para cada n 1,
()
bn = 2,1 an1 + 2,2 bn1 ,
umeros reales; ademas tendremos unas condiciones iniciales a0 = p
donde los ij son ciertos n
y b0 = q. Resulta conveniente escribir este sistema en forma matricial:
 



1,1 1,2
an1
an
=
para n 1,
()
bn
2,1 2,2
bn1



A
17
En la secci
on 10.4 aprenderemos a resolver sistemas de ecuaciones de recurrencia con otras herramientas
(funciones generatrices) que, en principio, permiten abordar sistemas de ecuaciones de
ordenes mayores que 1.

(versi
on preliminar 23 de octubre de 2008)

6.2. Resolucion de ecuaciones de recurrencia

379

Llamaremos matriz de transici


on18 a la matriz A. Buscamos dos sucesiones (an ) y (bn )
que satisfagan las ecuaciones y las dos condiciones iniciales a0 = p y a1 = q.
La resoluci
on es sencilla y directa: aplicamos reiteradamente la relacion () hasta llegar
al caso n = 0, que conocemos (de manera analoga a lo que hacamos cuando se trataba de
una u
nica ecuaci
on de primer orden). En el lenguaje matricial, la iteraci
on de la regla de
recurrencia se traduce, simplemente, en la multiplicaci
on de matrices:
 

 
2


n


1,1 1,2
an1
1,1 1,2
an2
1,1 1,2
a0
an
=
=
= =
.
bn
2,1 2,2
bn1
2,1 2,2
bn2
2,1 2,2
b0
Como conocemos los valores de a0 y b0 , todo el problema se reduce al de calcular la potencia
n-esima de la matriz de transicion A. Es esto sencillo? La respuesta es casi obvia: depende
(de como sea la matriz A, claro). Veamos un par de ejemplos primero.
Ejemplo 6.2.6 Supongamos que tenemos un sistema de ecuaciones

an = 2 an1 ,
bn = 3 bn1 ,
para cada n 1, junto con las condiciones iniciales a0 = 7 y b0 = 5.
El sistema es muy especial: las dos ecuaciones estan desacopladas (para resolver una ecuacion no necesitamos la informaci
on de la otra). As que podramos optar por resolver cada
ecuacion por su lado. Pero tambien podemos seguir el procedimiento general:
 
n 


2 0
a0
an
=
.
bn
b0
0 3
As que hay que calcular la potencia n-esima de una matriz diagonal. El lector comprobar
a sin
dicultad que


n  n
2
0
2 0
=
.
0 3n
0 3
As que

an
bn


=

2n 0
0 3n



a0
b0


=

2n a0
3n b0


=

7 2n
5 3n


.

Leyendo componente a componente, obtenemos las soluciones de cada ecuacion.

Ejemplo 6.2.7 Consideremos ahora las condiciones iniciales a0 = 1, b0 = 3 y el sistema



an = 2 an1
para cada n 1.
bn = an1 + 2 bn1
La soluci
on del problema viene dada (escrita en forma matricial) por
 
n  

2 0
1
an
=
.
bn
1 2
3
18

El nombre viene sugerido por la siguiente interpretaci


on: tenemos un cierto sistema descrito por los valores
omo evoluciona el sistema de un instante
de las sucesiones an y bn en cada tiempo n. La matriz A nos dice c
de tiempo al siguiente, cu
al es la regla de transici
on entre ambos estados.

(versi
on preliminar 23 de octubre de 2008)

380

Captulo 6. Ecuaciones de recurrencia

Aunque la matriz no es diagonal, su forma es muy especial. No es muy difcil convencerse


(calculando los primeros casos y luego utilizando inducci
on) de que


n 
2n
0
2 0
,
=
n 2n1 2n
1 2
de manera que la soluci
on del problema es

 
  

0
an
2n
1
2n
=
=
n 2n1 2n
bn
n 2n1 + 3 2n
3
Compruebese que, efectivamente, las sucesiones an = 2n y bn = n 2n1 + 3 2n verican el
sistema de ecuaciones y las condiciones iniciales.

Hemos visto que hay dos casos especialmente sencillos: cuando la matriz es diagonal,
 n




0
0
n
,
A=
, porque entonces A =
0 n
0
o cuando la matriz es de la forma


0
A=
,
1


An =

en cuyo caso

n
0
n n1 n


.

En general, la matriz de transici


on A no sera tan sencilla. Pero hemos destacado estos dos
casos porque nos van a permitir resolver el caso general. Si para ciertos n
umeros y y para
cierta matriz P (2 2 e invertible) pudieramos escribir la matriz A de la siguiente manera:


0
A=P
P 1 ,
0
entonces el calculo de An sera inmediato. Vease, por ejemplo, el caso de la potencia 2:

2

A = P

0
0


P

2
=P

0
0


P


P

0
0


P


P

2 0
0 2

P 1 ,

donde hemos utilizado que P P 1 = I, la matriz identidad 2 2. Un argumento similar (o


una prueba por inducci
on) nos permite deducir que
 n


0
n
P 1 ,
A =P
0 n
con lo que el problema se resolvera efectuando este producto de tres matrices (recordemos
que al nal habr
a que multiplicar tambien por el vector de condiciones iniciales).
Lo mismo ocurrira si A se pudiera escribir como




n
0
0
1
n
P 1 .
A=P
P , en cuyo caso tendramos que A = P
n n1 n
1
(versi
on preliminar 23 de octubre de 2008)

6.2. Resolucion de ecuaciones de recurrencia

381

Al lector que haya pasado por un curso de Algebra


lineal b
asica todo esto le resultara familiar. Gran parte del esfuerzo que se emplea en uno de esos cursos va dirigido al problema
de la diagonalizaci
on o, en su caso, de la obtencion de formas can
onicas. Una de sus mejores
aplicaciones la encontramos, precisamente, en el calculo de potencias de una matriz.
El resultado basico sobre estas cuestiones asegura que, dada una matriz A de dimensiones 2 2, podremos escribirla de una de las dos siguientes formas19 :




0
0
1
o bien
A=Q
Q1 ,
A=P
P ,
1
0
donde los n
umeros y son los llamados autovalores de la matriz (en el segundo caso,
sera un autovalor doble), y la matriz P (
o Q) es invertible. Estas matrices se calculan a partir
de los autovectores asociados a los autovalores. Todo esto forma parte del folklore habitual

de los cursos de Algebra


lineal, as que no entraremos en los detalles y nos limitaremos a ver
un par ejemplos, en los que supondremos cierto manejo con estos conceptos.
Ejemplo 6.2.8 Una reacci
on en cadena: en un cierto medio hay a
tomos de un determinado elemento, contra los que lanzamos dos tipos de partculas, A y B. Cuando una partcula
A choca con un
atomo, este se desintegra, dando lugar a 2 del tipo A y a 3 del tipo B. Si es
una partcula B la que choca, entonces se producen 1 de A y 4 de B. Interesa el n
umero de
partculas de cada clase tras n unidades de tiempo, si partimos de 2 de tipo A y 1 de tipo B.
umero de partculas de tipo A y B, respectivamente, tras n unidades
Llamemos an y bn al n
de tiempo. Los procesos de desintegracion se traducen en el sistema de recurrencias

an = 2 an1 + bn1 ,
para cada n 1,
bn = 3 an1 + 4 bn1 ,
mientras que la condici
on inicial es a0 = 2 y b0 = 1. En terminos matriciales
 



 
n  

2 1
an1
an
2 1
2
an
=
, cuya soluci
on es
=
,
bn
bn1
bn
3 4
3 4
1
para cada n 1. As que, si llamamos M a la matriz de transici
on del sistema, solo resta
calcular su potencia n, para lo que ser
a necesario diagonalizarla. Empecemos calculando sus
autovalores, las soluciones de la ecuaci
on


2
1
det
= 0,
es decir, (2 )(4 ) 3 = 0,
3
4
ucleo de la
que resultan ser 1 = 1 y 2 = 5. El subespacio asociado al autovalor 1 es el n
aplicaci
on representada por la matriz M 1 I. As que debemos considerar el sistema

   
1 1
x
0
=
,
3 3
y
0
que nos dice que podemos tomar como autovector asociado al autovalor 1 = 1 a v1 = (1, 1).
19

Llamadas formas can


onicas de Jordan. En realidad, estamos obviando algunas sutilezas, porque lo expuesto es cierto si permitimos que los autovalores sean n
umeros complejos. Si no son reales, hay otras formas
can
onicas que nos permiten escribir la matriz en terminos reales.

(versi
on preliminar 23 de octubre de 2008)

382

Captulo 6. Ecuaciones de recurrencia

ucleo de M
El subespacio asociado al autovalor 2 = 5 viene dado por el n
si procedemos de manera analoga, obtenemos un posible autovector, v2 = (1, 3)
autovalor. Ya tenemos la matriz P de cambio de base,





1 3 1
1
1 1
1
y por tanto M = P
P =
, cuya inversa es P =
1
1
0
1 3
4

2 I, y
para este
0
5

P 1 .

Calcular entonces M n ya no representa dicultad alguna:



 n




1
1
1 1
1
3 + 5n
0
1 + 5n
3 1
n
=
.
M =
0 5n
1 1
4 1 3
4 3 + 3 5n 1 + 3 5n
Finalmente, la soluci
on del problema es



 


1
1
3 + 5n
5 + 3 5n
1 + 5n
2
an
=
=
.
bn
1
4 3 + 3 5n 1 + 3 5n
4 5 + 9 5n
As que, en tiempo n, hay an = 14 (5 + 3 5n )
partculas de tipo A, mientras que de tipo B
Tipo A 2 5 20 95
470 2345
habr
a bn = 14 (5 + 9 5n ). A la izquierda listaTipo B 1 10 55 280 1405 7030
mos los primeros casos. Ambas poblaciones crecen muy r
apidamente, como corresponde al caracter exponencial de las soluciones. Y, como
la propia din
amica del sistema sugera, va a haber m
as partculas del tipo B que del tipo A.
Pero podra interesarnos saber cual es la razon entre las dos poblaciones, si se aproxima
a un lmite nito o no. Observese que, para n muy grande, an 34 5n y bn 94 5n , por lo que
la raz
on entre las poblaciones se aproxima (y bastante r
apidamente) a una proporci
on 1:3.
Esto ya no era tan obvio con s
olo mirar las recurrencias!

tiempo

0 0 0 0
En el caso de dimension 2, la forma de Jordan no diagonal apa1 0 0 0 
rece cuando la matriz A tiene un autovalor doble , y el n
ucleo de 


 0 1 0 0 

A I solo tiene dimensi
on 1 (s
olo hay un autovector asociado). Si 
 . . . .
.. .. 
.
.
.
.

. . . 
consideramos matrices de dimension d, la descripci
on se complica  . . .

 0 0 0 0 
un poco: la forma de la matriz can
onica de Jordan depender
a de la
0 0 0 1
multiplicidad de los autovalores y del n
umero de autovectores que
tenga asociado cada uno de ellos. Pero, esencialmente, es una matriz diagonal por cajas,
donde cada una de las cajas es de la forma que aparece a la derecha. Veamos un ejemplo:

Ejemplo 6.2.9 Consideremos el siguiente

an =
bn =
a0 = 1, b0 = 3, c0 = 2 y

cn =

problema:
2 an1 + bn1 + cn1
3 bn1
bn1 + 3 cn1

para cada n 1.

La soluci
on del sistema viene dada por la ecuaci
on a 2
n
matricial que escribimos a la derecha. Los autovalores bn = 0
de A (las soluciones de det(A I) = 0) son los n
umeros
cn
0
2 y 3 (este, un autovalor doble). Para el autovalor simple, = 2, el calculo del n
ucleo de A 2 I nos dice que podemos tomar
(versi
on preliminar 23 de octubre de 2008)

1
3
1

n
1
1
0 3
3
2

como autovector

6.2. Resolucion de ecuaciones de recurrencia

383

on 1.
asociado a v1 = (1, 0, 0). Pero el subespacio asociado al autovalor 3 solo tiene dimensi
Concretamente, el n
ucleo de A 3 I esta generado por el vector (1, 0, 1). La matriz can
onica
no va a ser diagonal, as que tendremos que recurrir a un sustituto. Calculamos el n
ucleo de
2
la matriz (A 3 I) , que resulta estar generado por los vectores (1, 0, 1) y (0, 1, 0). Tomamos
ahora un vector que este en el n
ucleo de (A 3 I)2 pero no en el de A 3 I; v2 = (1, 1, 1)
podra valer. Ahora calculamos
v3t = (A 3 I) v2t ,
para obtener v3 = (1, 0, 1). Estos tres vectores forman (al ponerlos como columnas) la matriz P que mostramos a continuaci
on, junto con su inversa P 1 :

1
P = 0
0

1 1
1
1 0 , P 1 = 0
1 1
0

0 1
1
0
1 1

de manera que

2 0
A=P 0 3
0 1

0
0 P 1 .
3

El c
alculo de An es ahora ya muy sencillo:

2n
n
A =P 0
0

0
3n
n 3n1

n
0
2
0 P 1 = 0
0
3n

n 3n1
3n
n 3n1

3n 2n

0
3n

Finalmente, la soluci
on del sistema se obtiene multiplicando M n por el vector de condiciones

iniciales, y el resultado es an = 2n + (n + 2) 3n , bn = 3n+1 y cn = (n + 2) 3n .


6.2
EJERCICIOS DE LA SECCION
6.2.1 Consideramos la ecuaci
on de recurrencia xn = xn1 + , para cada n 1, junto con el valor
inicial x0 . Compruebese que
(a) si = 0, entonces xn = n x0 para cada n 0;
(b) si = 1, entonces xn = n + x0 para cada n 0;
(c) si = 1, entonces

n

xn = x0 +

n 1
1


,

para cada n 0.

on de la
6.2.2 Consideremos dos sucesiones (an ) y (bn ) linealmente independientes y que sean soluci
ecuaci
on an = an1 + an2 para cada n 2. Sea (cn ) otra soluci
on de la ecuaci
on. Compruebese
on lineal de las sucesiones (an ) y (bn ).
que la sucesi
on (cn ) puede escribirse como combinaci
6.2.3 Estamos con la ecuaci
on an = an1 + an2 , para cada n 2, junto con las condiciones inion caracterstica
ciales a0 y a1 . Suponemos que r y s son las dos races (reales y distintas) de la ecuaci
asociada. Compruebese que la soluci
on de la ecuaci
on de recurrencias (y las condiciones iniciales) se
puede escribir como
r n sn
+ a 0 sn .
(a1 a0 s)
rs
Ahora suponemos que r y s se hacen cada vez m
as pr
oximas (en el lmite, estaremos en el caso de
una raz doble de la ecuaci
on caracterstica). Compruebese, quiz
as apelando a la noci
on de derivada,
que la soluci
on en el caso de raz doble se puede escribir como combinaci
on lineal de sn y nsn .

(versi
on preliminar 23 de octubre de 2008)

384

Captulo 6. Ecuaciones de recurrencia

6.2.4 Consideremos la ecuaci


on inhomogenea
()

an an1 an2 = f (n)

y la homogenea asociada

()

an an1 an2 = 0 .

(a) La suma de dos soluciones de () no es soluci


on de (), mientras que la suma de dos soluciones de
() s que es soluci
on de (). Compruebese que la resta de dos soluciones de () es soluci
on de ().
(b) Compruebese que toda soluci
on de () se puede escribir como la suma de la soluci
on general de ()
con una soluci
on particular de ().
6.2.5 (a) H
allese una f
ormula para los n
umeros un que verifican: u0 = a, u1 = b y un+2 = un + n
para cada n 1.
(b) Un triple lineal es una lista de 3 n
umeros naturales a, b, c, ordenados, a < b < c, y tales que
umero de triples lineales formados con n
umeros comprendidos entre 1 y n.
c b = b a. Sea Tn el n
Pruebese que
T2n+1 = T2n + n
y obtengase una recurrencia an
aloga para T2n . H
allese una f
ormula para Tn .
6.2.6 Resuelvanse las recurrencias:
(a) an+3 = 4an+2 5an+1 + 2an , n 0 con condiciones iniciales a0 = a1 = 0 y a2 = 1.
(b) an+3 = 3an+2 3an+1 + an , n 0 con condiciones iniciales a0 = a1 = 0 y a2 = 1.
(c) an+2 = 2an+1 + 2an + n, n 0 con condiciones iniciales a0 = 0 y a1 = 2.
6.2.7 En un juego en el que p = 0,49, hacemos apuestas de 1 ficha y nos retiramos cuando tengamos
100 fichas. Que cantidad inicial de fichas deberemos tener para poder asegurar que tenemos, al menos,
un 50 % de oportunidades de terminar ganando?
6.2.8 H
allense las sucesiones (an ) y (bn ) dadas por a0 = 1, b0 = 0 y


2an = 5an1 + bn1


2bn = an1 + 5bn1

6.2.9 Consideremos la figura que dibujamos a la derecha. Colocamos una ficha (aleatoriamente) en una de las posiciones a, b
o c y luego la movemos
siguiendo los caminos posibles del dibujo (todos los movimientos son igualmente probables). Llamamos an a la probabilidad de que la ficha este en a tras n
movimientos (de forma an
aloga se definen bn y cn ).
(a) Muestrese que: an + bn + cn = 1,

n = 0, 1, 2 . . .

(b) Calc
ulense an , bn y cn . Estmense estas probabilidades en el lmite n .
(c) C
omo cambiara el problema si la distribuci
on inicial no fuera aleatoria? Por ejemplo, si sabemos
que la ficha est
a inicialmente en a.
6.2.10 La sucesi
on (an ) est
a definida por una ecuaci
on de recurrencia lineal y homogenea de grado m.
Sea r el m
aximo m
odulo de las races de la ecuaci
on caracterstica correspondiente. Pruebese que
lm sup
n

|an |
< + .
rn

(versi
on preliminar 23 de octubre de 2008)

También podría gustarte